You are on page 1of 110
ENGLISH PROFICIENCY TRAINING (for use only by students of Sriwijaya University) By: Bambang A. Loeneto Alhenri Wijaya Dedi Kurniawan Lingga Suganda Zuraida LANGUAGE TECHNICAL IMPLEMENTATION UNIT SRIWIJAYA UNIVERSITY 2019 PREFACE* _ The lessons provide you with some effective explanations and helpful studying techniques, particularly when you take a test consisting of listening comprehension, structure, and reading comprehension.’ However, maximum result will be achieved if you study on your own intensively. Follow the following steps. : Step 1: Set your target score Remember that some tests like TOEFL-Test of English as a Foreign Language, IELTS-International English Language Testing. System, or SULIET-Sriwijaya. University Language Institute English Test are standardized tests to measure your ability to use and understand the-English language as it's read, written, heard and spoken in the university classroom and the real world. Sriwijaya University has its own score (SULIET) ‘requirement for students to graduate. Therefore, you have to find out the score required by your study program or department. You can start setting your score: target this way. Research a bit on your chosen faculty and find out the minimum score. This way, you have a basis on how broad and how intense your studies need to be on the long run. Of course,.a desirable score is a score that exceeds the minimum. Step.2: Gather all essential studying materials. A good preparation makes all the difference—and a good: preparation requires the right studying materials. Consequently, equip yourself with other learning materials to achieve maximum results. Step 3: Make a studying plan With all the materials you need to go through, you have to have a studying plan. This way, you are able to have all your studying time broken down into tidbits and organized in.a schedule you can follow up. There is no "correct" or “one-size-fits-all” study plan. When creating your personalized studying plan, you should ask yourself and analyze: Which subjects do | need to Prioritize? How many free hours are available? Step 4: Familiarize yourself with the test format Once you've built a good foundation..on. all four skills—reading comprehension, academic level vocabulary, standard grammar—the next crucial step is to familiarize yourself with the test format. Your studies will be a lot simpler and effective once you get the idea of the content of the test, the question and task types of each section, and also their rules and regulations. The best way to do this is by regularly taking the practice test. Step 5: Join a Course or Hire an Expert The more you take practice tests, the more you'll able to-identify: your weak spots. In this case, for a faster and more effective progress, it's best to have a tutor alongside your studying journey. Palembang 2019 BAL-AW-DK-Z-LA TABLE OF CONTENTS. PREFACE.. TABLE OF CONTENTS. LISTENING SECTION... LESSON 1 Question Response (Part 1) 1. Identifying Time 2. Identifying Peopl 3. Identifying an Opinion 4. Identifying a Choice .. LESSON 2 Question Response (Part 2) 1. Identifying a Suggestion’. 2. Identifying a Reason 3. Identifying a Location 4. Strategy Review. 3. Identifying a Reason. 4. Identifying an Opinio1 LESSON 4 Talks. 4. Identifying the Audience .. 2. Identifying the Situation 3. Identifying Topics... 4. Identifying a Request STRUCTURE SECTION LESSON 1 Incomplete Independent Clauses A. Clauses ... B. Missing Subjects, Verbs, Objects, and Complements .. C. Clauses with There and It LESSON 2 Incomplete Adjective Clauses LESSON 3 Incomplete or Missing Participial Phrases LESSON 4 Incomplete or Missing Appositives Incomplete Adverb Clauses LESSON 5 Incomplete Noun Clauses LESSON 6 Missing or Incomplete Prepositional Phrase: Practice Test READING SECTION .. LESSON 1 Topic and Stated Main Idea.. LESSON 2 Implied Main Ide: LESSON 3 Making Inferencing LESSON 4 Pronoun Referents Reading Comprehension Pra LISTENING SECTION In this section, you will have an opportunity to demonstrate your ability to understand conversations and talks in English. Study each part and practice a lot with other similar learning materials you can have from other sources. Tips: 1. Do not panic. Concentrate on the current question only, and do not think about how you answered other questions. This is a habit that can be learned through practice. 2. Avoid spending too much time on any one question. If you have given the question some thought and you still don't know the answer, eliminate as many answer choices as possible and then select the best choice. You can check your responses after all the questions have been answered so you can stay focused and save time. 3. Pace yourself so you have enough time to answer every question. Be aware of the time limit for each section/task, and budget enough time for each question/task so you do not have to rush at the end. It is a good idea to check the clock periodically to monitor progress. vei) Listening or LESSON 1 QUESTION RESPONSE (Part 1) In this part you will leam how certain words.will help you: identify.the purpose of a question. These are the purposes you will leam about: identifying time identifying people identifying an opinion identifying a choice 1. IDENTIFYING TIME . You will hear questions that ‘ask ‘about time. Sore questions will begin with When or How jong. Others will be yes/no questions. The answer to a-yes/no question is sometimes a statement without yes or no. Example 1 When did she leave for work? (A) About an hour ago. (B) It doesn't work. (C) As soon as he's ready. ‘The correct answer is (A). Choice (B) tries to confuse you by repeating the word work. Choice (C) tries to confuse you by changing the tense to present and the person to he. : Example 2 Haven't you filled out the application yet? (A) They filled the jet with fuel. (B) I've been too busy. (C) | applied it over the surface. ‘The correct answer is (B). Note the no is implied in the response. The person was too busy to take the time to complete the application. Choice (A) tries to confuse you by repeating the word filled and using the similar-sounding word jet for yet. Choice (C) tries to confuse you by using applied with a different meaning. Example 3 sate ‘We'll leave at 5, so try to be on time. (A) That leaves five of us. -(B) I'm never late.'I'll be there at 4:59., (C) The plane is on time. The correct answer is (B). The speaker suggests that the listener will not be on time; but ‘the listener responds that he/she is-never late:and will in:factbe there one. minute’early. Choice (A) tries to confuse you by repeating the word leave: but..with:a different meaning. The word five is repeated, but here it means people ne time . day. Choice (C) repeats the phrase on time but in a different context. These are some words you. might hear. in‘questions about time: when early how long morning, noon, afternoon, evening, night what time at 1:00, 2:30 yet today, this week, this month, this year: still yesterday, last week, last — fast baa 1 on time tomorrow late Practice: Identifying. Time . Directions: Listen to the questions, which are followed by three responses. They will not be written out for you. Choose the best response to each queation wr ° fofa| wlolal/ololololo/o/ofo elolole[efele 2. IDENTIFYING PEOPLE You will hear questions that ask about people. Some questions will begin with Who or Whose. Others will be yes/no questions. The answer to a yes/no question is sometimes a statement without yes or no. Example 1 ‘Who's :responsible for making the bank deposits? (A) Ms. Rotelli always makes the deposits. (B).We'll send our response soon. (C) We use the National Bank. The correct answer is (A). Choice (B) tries to confuse you with the similar-sounding word response for responsible. Choice (C) tries to confuse you by repeating the word bank. Example 2 Are you in charge of this project? . (A) No, they only.charged us 100 dollars. (B) No, Mrs. Ono is the supervisor. ({C) No, it's not very large. ‘The correct answer is (B). Choice (A) tries to confuse you by using:charged but with a different meaning. Choice (C) tries to confuse you with the similar-sounding word large for charge. i‘ Example 3 | called the contractor to repair the leak. (A) You should have called a plumber. (B) His contract is due next week. (C) The telephone repair person contacted me. The correct answer is (A). A plumber can repair a leaking pipe. Choice (8) tries to confuse you by using the word contract with a different meaning and using the similar- sounding word week for leak. Choice (C) tries to confuse you by using the associated word telephone for called and the similar-sounding word contacted for contractor The word repair is repeated. These are some words you might hear in questions about people: who who's: ‘an occupation title whose name Practice: Identifying People’ * ' Directions: Listen to the following questions, which are followed BY three responses. They will not be written out for you. Choose the best response to each question. oO/@|N[ ala} slofrn] a ° Lo 3. IDENTIFYING AN OPINION... ‘ i You will hear questions that askSdirigone’s opinion. ‘Somé questions will begin with What or How. Others will be yes/no questions. The answer to a yes/no question is sometimes a statement without yes or no. Example 1 What did you think of the movie? (A) I liked it a lot. (B) | saw it yesterday. - (C) I moved the furniture myself: The correct answer is (A). Choice (B) tries to confuse you by giving an inappropriate response to an opinion question. Choice (C) tries to confuse you with the similar- sounding word moved for movie. F Example 2 Do you think we need to hire more people? (A) Yes, I'd like to hear more about it. (B) Yes, we need a few more employees. (C) Yes, prices are getting higher. The correct answer is (B). Choice (A) tries to confuse you, with the similar-sounding word hear for hire. Choice (C) tries to confuse you with higher, which sounds the same as hire. Example 3 loved this book. (A) | like to cook, too. (B) Book me a ticket, please. (C) I didn't think it was so great. The correct answer is (C). The listener does not agree with the speaker. Choice (A) uses the associated word like for love and the similar-sounding word cook for book. Choice (B) repeats the word book but with a different meaning and as a verb, not a noun. These are some words you might hear in questions about an opinion: what believe how your opinion why like/didn't like because love think Practice: Identifying an Opinion Directions: i Listen to the questions, which are followed by three responises. They will not be written out for you. Choose the best response to each question. TAT BG 7 1fa[sBic 2/[a|B|c. 3{alBlc 4falei[c s/alBic 6 | A.) B.C] 7lalBlc alaleic 9 [ads fc rofals lc] 4. IDENTIFYING A CHOICE You will hear questions that give someone a choice. Some questions will begin with What or other wh-question words. Others will be. yes/no questions. The answer to a yes/no question is sometimes a statement without yes or no. These questions usually will have two choices joined by or. Example 1 Which does that author write more of, poems or essays? (A) She almost always writes poems. (B) | read those poems yesterday. (C) She owns two homes. The correct answer is (A). Choice (B) tries to confuse you by repeating the word poems. Choice (C) tries to confuse you with the similar-sounding word homes for poems. Example 2 Do you prefer yellow or blue? (A) Yes, I do. (B) He's a very nice fellow. (C) Blue's my favorite color. The correct answer is (C). Choice (A) tries to confuse you by giving an inappropriate response to a choice question. Choice (C) tries to confuse you with the similar- sounding word fellow for yellow. Example 3 \ can't decide between the morning flight or the afternoon one. (A) We took a ride yesterday aftemoon. (B) Go before noon. It's less crowded. (C) There are more flies at noon. The correct answer is (B). The listener gives the speaker a reason to choose the momning flight. Choice (A) tries to confuse you by using the similar-sounding word ride for decide. Choice (C) uses the similar-sounding words more for moming, flies for flight, and noon for afternoon. These are some words you might hear in questions that give someone a choice: what or rather which prefer Practice: Identifying a Choice Directions: Listen to the questions, which are followed by three responses. They will not be written out for you. Choose the best response to each question. ol/@|nlolala}olr)= Sl >| >| >] >| >) >] >] >] >] > o\o Do w/o\o o/o\o w\o O/A/O/al/OloO}alolalola 5 LESSON 2 QUESTION RESPONSE (Part 2) In this chapter you will learn how certain words will help you identify the purpose of a question. These are the purposes you will learn about: identifying a suggestion identifying a reason identifying a location 1, IDENTIFYING A SUGGESTION You will hear questions that give a suggestion. Some questions will begin with Why or Let's. Others will be yes/no questions. The answer to a yes/no question is sometimes a statement without yes or no. Most of the questions that give a suggestion are yes/no questions. Example 1 Why don't we take a break? (A) That sounds like a good idea. (B) It didn't break. (C) This is very good cake. The correct answer is (A). Choice (B) tries to confuse you by using the word break with a different meaning. Choice (C) tries to confuse you by using the similar- sounding word cake for break. Example 2 Can | get you something to eat? (A) Yes, | picked up something. (B) Thank you. That's very kind of you. (C) We ate everything on the table. The correct answer is (B). Choices (A) and (C) try to confuse you by incorrectly answering a present tense question with a past tense answer. Example 3 Let's not take a taxi. (A) Yes, I'd rather walk. (B) The tax is included. (C) | wrote a note to Tashi. The correct answer is (A). The speaker made the suggestion not to take a taxi and the listener agreed. Choice (B) tries to confuse you by using the similar-sounding word tax for taxi. Choice (C) tries to confuse you by using the ‘similar-sounding phrase note to with not take and taxi with Tashi. These are some words you might hear in questions giving a suggestion: why don't we you should what about how about let's why don't ought to Practice: Identifying a Suggestion Directions: Listen to the questions or statements, which are followed by three responses. They will not be written out for you. Choose the best response to each question A|BIC 1|/A] B/C 2/A|BiC 3/A|BIiC 4{/A/|B/C |}5}/A/B]C 6)/A|BIC 7{A/BIC 8{/A|BIC 9|A{BiC 10; A|BIC 2. IDENTIFYING A REASON You will hear questions that ask for a reason. Some questions will begin with Why. Others will be yesino questions. The answer to a yes/no question is sometimes a statement without yes or no. Example 1 Why didn't you make the dinner reservation? (A) | reserved my hotel room. (B) | eat dinner at six. (C) | forgot the name of the restaurant. The correct answer is (C). Choice (A) tries to confuse —_ you by using the word reserved in a different context. Choice (B) tries to confuse you by repeating the word dinner. 10 Example 2 Aren't you working late again tonight? (A) No, | have to go home early. (B) | left at eight o'clock. (C) Wait until tomorrow night. The correct answer is (A). Choice (B) tries to confuse you by answering incorrectly with the past tense. Choice (C) tries to confuse you by using the similar-sounding words wait for late and night for tonight. Example 3 | can't drive without my glasses. (A) Can you dive, but not swim? (B) The glasses are full of water. (Cc) And | don't have a license. The correct answer is (C). The reason the speaker can't drive is he doesn't have his eyeglasses. The listener can't drive because he doesn't have’ a driver's license. Choice (A) uses the similar-sounding word dive for drive. Choice (B) uses water glasses instead of eyeglasses. These are some words you might heai in questions asking for a reason: why excuse why didn't reason Practice: Identifying Reason Directions: Listen to the questions or statements, which are followed by three responses. They will not be written out for you. Choose the best response to each question. “ERTS EC] 1} A]B LC 2);A/BiC 3}AIlBIC 4|/A/B/C S;A/BIC 6}/A/BIC 7TIA{[BIC BIA;B/C 9}/AIBIC 10} A/B/C 4 3. IDENTIFYING A LOCATION You will hear questions that ask about a location. Some questions will begin with What or Where. Others will be yes/no questions. The answer to a yes/no question is sometimes a statement without yes or no. Example 1 What about moving the desk next to the door? (A) There isn't enough room. (B) | didn't open the door. (C) He's next in line. The correct answer is (A). Choices (B) and (C) try to confuse you by repeating the words door and next. Example 2 Can you tell me how to get to the post office? (A) OK, I'll wait in your office. (B) Yes, get me some stamps. (C) Sure. Go to the corner and take a right. The correct answer is (C). Choice (A) tries to confuse you by repeating the word office. Choice (B) tries to confuse you by using the word get but with a different meaning. Example 3 | left my pen on your desk. (A) My dogs are in the pen. (B) | put it in the drawer. (C) No, I left at eleven. The correct answer is (B). The location of the pen moved from the desk to the desk drawer. Choice (A) repeats pen but with a different meaning. Choice (C) repeats the word left with a different meaning and uses the similar-sounding word eleven for pen. These are some words you might hear in questions asking about a location. what under behind where over at right how far near left next to far names of places beside by 12 Practice: Identifying a Location Directions: Listen to the questions, which are followed by three responses. They will not be written out for you. Choose the best response to-each question. Pea PB} Ce 1{/A;BIC 2/A/BiC 3/A/BIC 4)A] B/C S{A|BIC 6{/A/}BIC 7T{LA{BIC BPAIBIC 9/A/BIC 10;A}B}C 4. STRATEGY REVIEW: In the exercises for Part 2, you learned how certain words would help you identify the purpose of a question. Knowing these words will help you choose the right answer. Listen for words that identify: time, such as when, how long, what time, etc. people, such as who, whose, who's, etc. an opinion, such as what, how, why, etc. a choice, such as what, which, or, etc. a suggestion, such as why, let's, what about, etc. a reason, such as why, excuse, reason, etc. a location, such as what, where, how far, etc. In the exercises for Part 2, you saw how certain answer choices try to confuse you. Here are the ways that choices may seem correct to you. Some choices have words that sound similar. ‘Some choices repeat words. Some choices have words used in a different context. Some choices use incorrect verb tense or person. Some choices are an inappropriate response to the type of question. 13 Look at these examples: Example 1 Mary is never late for her meetings (A) He is always on time. (incorrect person) (B) Her meeting is running late: (words used in a different context) (C) She's very punctual. (correct answer) Example 2 Are you hiring an assistant or a secretary? (A) | need both. (correct answer) (B) Yes, | am. (inappropriate response to choice questions) (C) | was hired yesterday. (incorrect verb tense and person) Example 3 How can | get to the train station from here? (A) The radio station is a block away. (word used in a different context) (B) You can take the A-4 bus. (correct answer) (C) The plane leaves in ten minutes. (words that sound similar) Strategy Practice Directions: Listen to the questions, which are followed by three responses. They will not be written out-for you. Choose the best response to each question. Listen again and see if you can recognize how a choice tries to confuse you. Bic 3 >|>|>|>[>[>|>[>|>[>|>[>[>[>][>[>|>|>|>[>[>| 00} 03] 09] 09) co} 00] e9] co} 03} 00] co] c9} 09] ca} c0]c0| eo} 00] co} c0| eefelolalelelo[ala[alolole|a|olo|a|ofo} 14 LESSON 3 CONVERSATION In this lesson, you will learn how certain words will help you identify the purpose of a question in a conversation. These are the purposes you will learn about: identifying intent identifying the topic identifying a reason identifying an opinion 1. IDENTIFYING INTENT Alistening question of a conversation may ask about a person's intent to do something. For example: What will she do? What does she plan on buying? Read the questions and the answer choices quickly before you listen to the conversation. When you listen to the conversation, listen for answers to the questions about intent. Example 1 SPEAKER A: I'm going to the electronics store after lunch. Do you want to go? SPEAKER B: Sure. Are you getting something for your computer? SPEAKER A: No, | just need to pick up a new battery for my phone. What does Speaker A want to purchase? (A) Food. (B) Acomputer. (C) A battery. (D) Aphone. The correct answer is (C). Choice (A) uses the word food, which is associated with lunch but isn't what she's going to buy. Choice (B), computer, is mentioned in something for your computer, but she says that's not what she will buy. Choice (D), phone, is mentioned, but itis the battery, not the phone itself, that she will buy. 15 Example 2 SPEAKER A: Could you answer the phone for me this afternoon? SPEAKER B: Of course. Are you going to be in a meeting? SPEAKER A: No, | have to leave a little early to get to the bank before it closes. What does she want to do? (A) Buy clothes. (B) Goto the bank. (C) Gotoa meeting. (D) Answer the phones. The correct answer is (B). Choice (A) tries to confuse you with the similar-sounding word clothes for closes. Choice (C), go to a meeting, is mentioned, but she says that she isn't going to do that. Choice (D) is what she asks her colleague to do. Look for these words in a question that asks about intent: plan going to will probably Practice: Identifying Intent Directions: Listen to the conversation and then choose the statement that best answers the question. 1. What do the speakers plan to do this Sunday? (A) Watch a baseball game. (C) Go to a concert. (B) See a movie. (D) Visit the capital. 2. What will the man probably do? (A) Buy a new rug. (C) Paint the old furniture. (B) Buy new furniture. (D) Get another office. 3. What is the woman going to do? (A) Eat a pizza. (C) Put things in order. (B) Pick up her office. (D) Make a delivery. 4. What will the man probably have? (A) Some ice cream. (C) Some cold cuts. (B) Some hot tea. (D) Some iced tea. 5. What form of transportation will the speakers use? (A) Train. (C) Bus. (8) Walking. (D) Cab. 6. How will the woman pay? (A) With a money order (C) With cash. (B)With a credit card. (D) With a check. 16 7. What will the woman probably do? (A) Turn off the air-conditioning. (C) Turn on the air-conditioning. (B) Open the window. (D) Close the window. 8. What will the man do? (A) Buy a new machine. (C) Plug the machine in. (B) Fix the old machine. (D) Use his coworker's machine. 9. What does the woman want to do? (A) Look at the movie schedule. (C) Borrow some paper. (B) Read the news (D) Buy a newspaper. 410. What will the woman probably buy? (A)A sweater. (C)A skirt. (B)A suit. (D)A dress. 2. IDENTIFYING TOPIC Questions in a listening test for a conversation may ask about the topic. For example: What are they talking about? What is the problem? Read the questions and the answer choices quickly before you listen to the conversation. When you listen to the conversation, listen for answers to the questions about the topic. Example 1 SPEAKER A: Would you like more coffee? SPEAKER B: No! This coffee tastes terrible. Is the machine broken again? SPEAKER A: No, | think it's just dirty. No one ever cleans it. What is the problem? (A) The coffee is cold (B) The machine is dirty. (C) The machine is broken. (D) There isn't any more coffee e The correct answer is (B). Choice (A) repeats the word coffee, but the problem with the coffee is its taste, not its temperature. Choice (C) is mentioned as a possibility but is not the problem. Choice (D) tries to confuse you by repeating the word more Example 2 SPEAKER A: Give me your number and I'll call you later. SPEAKER B: OK. It's 555-1331. SPEAKER A: Is that home or office? What are they discussing? (A) A telephone number. (B) An address. (C) A letter. (D) An office. The correct answer is (A). Choice (B) uses address, which is associated with home. Choice (C) tries to confuse you with the similar-sounding word letter for later. Choice (D) tries to confuse you by repeating the word office. Look for these words in a question that asks about the topic: talking about discussing about Practice: Identifying the Topic Directions: Listen to the conversation and then choose the statement that best answers the question. 4. What is wrong with the car? (A) It has broken glass. (C) It has a flat tire. (B) It doesn't run fast. (D) It's out of gas. 2. What are the speakers talking about? (A)Acake. (B)Some steak. (C)Adiet. ' (D) The cook. 3. What is the problem with the restaurant? (A) It doesn't look nice. (C) It's too far away. (B) The service isn't good. (D) The food is bad. 4, What are the speakers discussing? (A) Airplane tickets. (C) A hotel reservation. (B) Movie tickets. (D) Books. 5. What is the lecture about? (A) How to speak in public. (C) How to buy a house. (B) How to save money. (D) How to live without a lot of money. 6. What are the speakers talking about? (A) Going to the movies. (C). Eating dinner (B). ATV show (D) A snowstorm. 18 7. Whats the problem with the hamburger? (A) It doesn't taste good. (B) It's stil in the kitchen. (C) It's undercooked. (D) It's bunt. 8. What are the speakers talking about? (A) Photocopy paper. (B) Money. (C) The newspaper. (D) Furniture. 9. What is the book about? (A) Earning money. (B) Managing your finances. (C) Finding a job. (D) Organizing your time. 10.What are the speakers discussing? (A) Going to work. (B) Going to a party. (C) Going out for dinner. (D) Going out for a drink. 3. IDENTIFYING A REASON A question in a listening for a conversation may ask about a reason for doing something. For example: Why Is he going? Why is she speaking softly? Read the questions and the answer choices quickly before you listen to the conversation. When you listen to the conversation, listen for answers to the questions about the reason. Example 1 ‘SPEAKER A: Maria, | hear you're moving away. Did you get a new job? SPEAKER B: NO, I'm going back to school. I'm going to get a degree in economics. SPEAKER A: Oh, yes. | hear the university in that city is very good. Why is Maria moving to a new city? (A) Because she got a new job. (B) Because she wants to study at the university. (C) Because the economy is bad. (D) Because her old city isn't very good. 19 The correct answer is (B). Choice (A) repeats the words a new job, but Maria says that is not the reason she is moving. Choice (C) tries to confuse you by using the word economy, which is similar to economics. Choice (D) tries to confuse you by repeating the words very good in a different context: Example 2 SPEAKER A: Look how late it is. I'll never get to the meeting on time. SPEAKER B: Well, here comes the bus now, so you're on your way. SPEAKER A: I'm still going to be late. Why is Speaker A upset? (A) It's time to go home. (B) He forgot to eat. (C) He has to take the bus. (D) He's going to arrive late. The correct answer is (D). Choice (A) tries to confuse you by using the word time in a different context. Choice (B) tries to confuse you by using the similar-sounding word cat for meeting. Choice (C), take the bus, is mentioned, but it is not the reason that Speaker A is upset. Look for this word in a question that asks about a reason: Why Practice: Identifying a Reason Directions: Listen to the conversation and then choose the statement that best answers the question. 1. Why are there no chairs? (A) The chairs haven't been ordered yet. (B) Nobody wants to sit down. (C) The chairs haven't arrived yet. (D) People prefer to sit on the floor. 2. Why will the man call the woman? (A) Because he needs some help. (B) To invite her to dinner. (C) Because he's bored. (D) To arrange a meeting. 3. Why is the woman going to Hawaii? (A) To spend her vacation. (B) To buy new clothes. (C) To attend a conference. (D) To visit friends. 20 4. Why did the woman arrive late? (A) She was in an accident. (B) She had a flat tire. (C) She felt tired. (D) She was waiting for someone. ad Why is the window closed? (A) It's cool outside. (B) The air-conditioning is on. (C) The room isn't warm enough. (D) The street is very noisy. ° Why doesn't the man want to take the elevator? (A) The elevator is slow. (B) They're going down. (C) It’s late. (D) He likes to walk. Nn Why is the man staying late at the office? (A) He has to finish his work. (B) He's expecting a phone call. (C) He has a day off tomorrow. (D) He isn't tired. 8. Why isn't the meeting in the conference room? (A) There aren't enough chairs. (B) It isn't big enough. (C) The office is more comfortable. (D) It's being painted. 9. Why does the woman suggest taking the subway? (A) The office is close. (B) A car is too fast. (C) Traffic is heavy. (D) It's late. 10. Why didn't the man eat lunch? (A) He forgot to eat. (B) He wasn't hungry. (C) He got to the cafeteria too late. (D) He didn't have time. 4. IDENTIFYING AN OPINION A question in a listening test for a conversation may ask about a speaker's opinion. For example: 24 What is her opinion about cooking? What does the speaker think about soccer? Read the questions and the answer choices quickly before you listen to the conversation. When you listen to the conversation, listen for answers to the questions about an opinion. Example 1 SPEAKER A: | think I'll like working with the new manager. He's very efficient. SPEAKER B: | agree with you, Max. And he's friendly, too. SPEAKER A: I'm sure he's the most experienced person in this office. What is Max's opinion of the manager? (A) He's agreeable. (B) He's friendly. (C) He's efficient. (D) He's inexperienced. The correct answer is (C). Choice (A) tries to confuse you by using agreeable, related to but different in meaning from agree. Choice (B) is the opinion of Max's friend. Choice (D) sounds similar to what Max said but actually has the opposite meaning. Example 2 SPEAKER A: What a movie. I've never laughed so hard. SPEAKER B: It really was awfully funny. SPEAKER A: | just love movies like that. What is said about the movie? (A) _ It was hard to understand. (B) _ It was funny. (C) twas awtul. (D) _ it was about love. The correct answer is (B). Choices (A), (C), and (D) try to confuse you by using the words hard, awful, and love but with different meanings. Look for these words in a question that asks about an opinion: think of opinion ‘say about Practice: Identifying an Opinion Directions: Listen to the conversation and then choose the statement that best answers the question. 1. What does the man think of the bus? (A) It's inconvenient. (B) It's relaxing. (C) It's too expensive. (0) It's fast. 2. What do the speakers say about Bob? (A) His work is good. (B) He's improving. (C) He talks too much. (D) He isn't doing a good job. 3. What is the speakers’ opinion of the hotel? (A) It's nice. (B) It isn't comfortable. (C) It's too big. (D) Its service could be better. 4. What do the speakers think of the weather? (A) It's too warm. (B) There's too much snow. (C) trains a tot. (D) _ It's too cold. 5. Whatdo the speakers say about TV? (A) _ It's boring. (8) There aren't many programs. (C) _ It's funny. (0) The programs are good 23° 6. What is the woman's opinion of the lecture? (A) _ Itwasn't enjoyable. (B) _ It was terrible. (C) _ It was interesting. (D) It wasn't long enough. 7. What does Jose say about his job? (A) _ It's important. (B) _ It's too far away. (C) It's difficult. (D) _ It's like his old job. 8. What does Sally think of her Spanish class? (A) _ She thinks it's too hard. (B) She likes it. (C) She thinks it's very easy. (D) She's having fun. 9. What do the speakers say about Bill? (A) He's lazy. (B) He's sick today. (C) He's usually late. (D) He's usually right. 10. What does the man think of the pizza? (A) It's not very good. (B) It's delicious. (C) It's terrible. (D) It’s too greasy 24 LESSON 4 TALKS In this lesson you will leam how certain words will help you identify the purpose of a question. These are the purposes you will learn about: identifying the audience identifying a situation identifying the topic identifying a request 1. IDENTIFYING THE AUDIENCE You will read questions that ask about audience. Read the question and the answer choices quickly before you listen to the talk. When you listen to the talk, listen for answers to questions about audience. Example: : Good service is the basis.of good business and will earn you good tips. Make sure the food you serve is prepared just as the customer requested it. Keep water glasses filled and remove dirty dishes as soon as the customer has finished eating. Who Is this talk directed to? (A) _ Restaurant customers. (8) Waiters. (C) Business owners. (D) Dishwashers. The correct answer is (B). Choice (A) repeats a detail of the talk that is not related to the answer. Choices (C) and (D) make incorrect inferences about details of the talk. Look for these words in a question that asks about audience: 25 Who directed to talking to Practice: Identifying the Audience DIRECTIONS: Listen to the talk and then choose the statement that best answers the question. 1. (A) (B) (A) (8) (A) (8) (A) (8) A) (8) (A) (8) (c) (0) Who is this talk directed to? Bookstore owners. (c) Professors. (D) Who is the speaker talking to? Store employees. (C) Customers. (D) Who is this message for? Ambulance drivers. (c) Patients. (0) Who is the speaker talking to? Airline pilots. (c) Flight attendants. (D) Who is this talk directed to? Radio station employees. (c) Public servants. (D) Who would call this telephone number? ‘Students. Economists Police officers. Bank tellers. Medical advisors. Office workers. Airplane passengers. People with'small children. Government officials. All city residents. People who want information about entertainment. People who want to hear a weather report. People who need jobs. People who need bus and train schedules. 26 7. Whois the speaker talking to? (A) Museum guards (C) City bus passengers. (8) Bus drivers. (D) Tourists. 8. Who is this announcement for? (A) _~ Auto mechanics. (C) Commuters. (B) Construction workers. (D) Airplane passengers. 9. Who is this advertisement directed to? (A) Employers. (C) Trainers. (8) Computer technicians. (D) Job seekers. 10. Who is this talk directed to? (A) People who want to become bankers. (B) People who want to buy a house. (C) People who want to work in an office. (D) People who want to go to the supermarket. 2. IDENTIFYING THE SITUATION You will read questions that ask about a situation. Read the question and the answer choices quickly before you listen to the short talk. When you listen to the talk, listen for answers to questions about situation. Example Thank you for calling Island Travel. If you'd like to book a place on our Hawaii tour, press 1. To make hotel or airplane reservations, press 2. Where would you hear this message? (A) Ata bookstore. (B) Atahotel. 27. (C)_ Onanaiplane. (D) Atatravel agency. The correct answer is (D). Choice (A) uses the word book with a different meaning. Choices (B) and (C) repeat details of the message that are not related to the correct answer. Look for this word in a question that asks about a situation: Where Practice: Identifying a Situation DIRECTIONS: Listen to the talk and then choose the statement that best answers the question. 1. Where would you hear this announcement? (A) Ata coffee shop. (C) Atan airport. (B) Ata grocery store. (D) Ata factory. rs Where is this announcement being made? (A) Onasubway. (C) Onabus. (B) Atthe airport. (D) Inataxi. 3. Where would you hear this announcement? (A) Ata bus station. (C) Atanightclub. (8) Ata school. (D) Ata soccer stadium. 4. Where is Martha going? (A) To the park. (C) Home. (B) Tothe gym. (D) Toarestaurant. 5. Where would you hear this announcement? (A) Onanelevator. (Cc) Inastore. (B) Ata theater. (0) Onabus. 28 6. Where can this talk be heard? (A) Inarestaurant. (C) Atamuseum. (8) Ina private home. (0) Atauniversity. 7. Where is the speaker? (A) Ata restaurant. (C) Ata theater. (B) Ata party. (D) Atahotel. 8. Where is this announcement being made? (A) Ata school. (C) Ata swimming pool. (8). Inagarden. (0) Ona farm. 9. Where can this announcement be heard? (A) Ata bookstore. (C) Atalibrary. (B) Atahotel. (D) Atanaccountant's office. 10. Where is Donna now? (A) Atthe office. (C) Atthe airport. (B) On the way to the office. (D) On the way to the airport. 3. IDENTIFYING TOPICS You will read questions that ask about topics. Read the question and the answer choices quickly before you: listen to the talk. When you listen to the talk, listen for answers to questions about the topic. Example Umbrella sales are sure to go up with all this rain we've been having. Rain continues all week. Saturday will be cloudy and breezy, and.the rain returns on Sunday. This is the time to invest in an umbrella company! 29 What is this announcement about? (A) Asale on umbrellas. (8) The weather. (C) Train schedules. (D) Stock market investments. The correct answer is (B). Choices (A) and (D) try to confuse you by repeating details that are not related to the question. Choice (C) uses the similar-sounding word train for rain. Look for these words in a question that asks about the topic: Topic purpose about talk about discussing kind Practice: Identifying Topics DIRECTIONS: Listen to the talk and then choose the statement that best answers the question. 1. What will Mr. Kim talk about? (A) Law. (C) Photography. (B) Retirement. (D) Traveling re ‘What is the topic of the meeting? (A) Office expenses. (C) _ Riding in taxis. (8) — Going out for lunch. (D) Changes in the office. 3. What is the purpose of this announcement? (A) To give the weather report. (8) To talk about traffic problems. (C) To announce that schools are closed. (D) To report the news. 30: (A) (B) (A) (B) (A) (8) (A) (8) (c) (D) A) (B) (A) (B) 10. (A) ®) What is the magazine about? Commercials. (c) Sports. (D) What kind of business is advertised? A conference planning service. (°c) Ahotel. (0) What is this announcement about? Weather. (Cc) Vacations. (0) What is the purpose of this talk? CD CD To explain why eating breakfast is important. To explain what to eat for breakfast. To explain when to eat breakfast. To explain who should eat breakfast. What kind of insurance is advertised? Health insurance. (C) Life insurance. (D) What is the purpose of this announcement? To report the news. (C) To explain the new schedule. (D) What is the topic of this report? Business sales. (C) Taxi fares. (D) 34 Television. News. A catering service. An entertainment business. Books. Mail. cD .GD Car insurance, Fire insurance. To introduce musicians. To discuss rock music. Elections. A tax increase. 4, IDENTIFYING A REQUEST You will read questions that ask about requests. Read the question and the answer choices quickly before you listen to the talk. When you listen to the talk, listen for answers to questions about requests. Example: The ABC Supermarket has openings for managers. Interested applicants should apply in person at 24 Riverdale Avenue on Saturday at 9:00 A.M. Bring three copies of your resume. Phone calls will not be accepted. How can you apply for this job? (A) Call the supermarket. (B) Send in aresume. (C) Goto 24 Riverdale Avenue. (0) Fill out an application. The correct answer is (C). Choices (A) and (B) repeat details of the announcement that are not correct. Choice (D) tries to confuse you by using the similar-sounding and related word application for applicant. Look for these words in a question that asks about a request: request ask how can Practice: Identifying a Request Directions: Listen to the talk and then choose the statement that best answers the question. 1. What are passengers asked to do? (A) Stand up. (C) Make a complete stop. (B) Stay seated. (D) Remain on the train. 32 (A) (8) A) (B) A) 8) (A) (B) A) (8) (c) (0) (A) (8) (A) (8) () (0) What are the members of the audience asked to do? Record the show. (C) Tum off their cell phones. Take pictures of the actors. (D) Wait in the lobby. How can you make an appointment with Mr.Schwartz? Wait for the beep. (C) Press one. Return the call. (D) Send an e-mail message. What are the passengers asked to do? Stand in line. (C) Show their passports. Pay by check. (D) Carry their own bags. What are drivers asked to do? Go downtown. (C) Use Constitution Avenue. Drive north on State Street. (D) Avoid accidents. How can callers speak to a customer service representative? Call the business line. Turn off the phone. Leave a message on the answering machine. Stay on the line. What are people asked to do? Send food and clothing. (C) Donate money. Leave their homes. (D) Go to Springfield. What are staff members asked to do? Give Mrs. Jackson some help. Ask the manager for assistance. Introduce themselves to Mrs. Jackson. Learn the office routine. 33 (A) (8) 10. (A) (B) What are staff members asked to do? Test the alarm. (C) Leave the building. Continue with their usual routine. (D) Avoid the elevator. What is the driver of the white car asked to do? Make a delivery. (C) Use the back entrance. Visit the building. (D) Move the car. STRUCTURE SECTION English sentences have various patterns. Some of them have just one subject and verb. However, many sentences in English have more than one clause. A clause is a group of words containing a subject and a verb. Whenever you you find a sentence on any English test with more than one clause, you need to make sure that every subject has a verb and every verb has a subject. Therefore, you should be familiar with these patterns. Tips: 1. Study the sentence by determining what is needed to complete the sentence correctly. Then study each answer according to how well it completes the sentence. Disregard any answers that do not complete the sentence correctly. . Be careful in trying to eliminate incorrect answers by just looking at the answers. The incorrect answers are generally correct by themselves. The incorrect answers are generally incorrect only when used to complete the sentence . Do not leave any answers blank. Be sure to answer each question even if you are not sure of the correct answer. 35 LESSON 1 INCOMPLETE INDEPENDENT CLAUSES A) Clauses All sentences consist of one or more clauses. A simple sentence consists of one clause. People need vitamins." The man took a vitamin pill. Judy lives in northern California. In the summer, Tom walks to his office. A compound sentence consists of two independent clauses joined by a coordinating conjunction (such as and and but). The man took a vitamin pill, and he drank a glass of orange juice. Judy lives in northern California now, but she was raised in Ohio. Acomplex sentence consists of an independent clause (called the main clause) and a dependent (subordinate) clause. Subordinate clauses may be adverb clauses, noun clauses, or adjective clauses. In the sentences below, the independent clauses are italicized. The man took a vitamin pill because he had a cold. (independent clause + adverb clause) ! didn't realize that Nancy was here. (independent clause + noun clause) Tom walks to his office, which is located on Broadway, every day during the summer. (independent clause + adjective clause) B) Missing Subjects, Verbs, Objects, and Complements All clauses have a subject and a verb. Clauses with an action verb often take a direct object as well. Subject Verb Object People need Vitamins. The verb’ missing from an independent clause may be a. single-word verb (need, was, took, had, walked) or a verb phrase consisting of one or more auxiliary verbs and a main verb (will need, has been, should take, would have had, had walked). The verbs may be active (need, take) or passive (was needed, is taken). 36 The missing subject and direct object may be a noun (people, vitamins, Tom) a noun phrase (some famous people, a vitamin pill, my friend Tom) or a pronoun. (He, she, it, and they are subject pronouns; him, her, it and them are object pronouns.) After the verb to be and certain other non-action verbs, a subject complement is used rather than a direct object. (Subject complements are also known as predicate nominatives and predicate adjectives.) Subject Verb Complement She. is an architect. The teacher seemed upset. Example: 01. The art of storytelling __ almost as old as humanity. (A) that is (B)is (C)itis (D) being 02.___ a few of the sounds produced by insects can be heard by humans. (A) Only (B) There are only (C) That only (D) With only 03.__ when lava cools very rapidly. (A) Because pumice is formed (B) To form pumice (C) Pumice is formed (0) Forming pumice 04. Duke Ellington wrote __ during his career. (A) that over a thousand songs (B) over a thousand songs (C) over a thousand songs were (D) there were over a thousand songs 05. Before the invention of the printing press, books (A) that were very rare (B) were very rarely (C) were very rare (D) as very rare 37 C) Clauses with There and It Some clauses begin with the introductory words there or it rather than with the ‘subject of the sentence. These introductory words are sometimes called expletives. : ‘The expletive there shows that someone or something exists, usually at a partie time or place. These sentences generally follow the pattern there + verb to be subject: There are many skyscrapers in New York City. There was a good movie on television last night. The expletive it is used in a number of different situations and patterns: It is important to be punctual for appointments. (with the verb to be + adjective + infinitive) Itwas in 1959 that Alaska became a state. (with the verb fo be + adverbial + noun clause) It takes a long time to learn a language. (with the verb fo take + time phrase + infinitive) - It was David who did most of the work. (with the verb fo be + noun + relative clause) Itand there, along with the verb and other sentence elements, may be missing from the stem. Example: 01. In Michigan, ___ over six hundred feet deep. (A) salt deposits (B) where salt deposits are (C) having salt deposits (D) there are salt deposits 02. __a tomato plant from 75 to 85 days to develop into a mature plant with ripe fruit. (A) It takes (B) To take (C) That takes (D) By taking Exercise: Choose the best answer. 4. In the United States, __ is generally the responsibility of municipal governments. (A) for water treatment (B) water treatment (C) where water treatment (D) in which water treatment 38 2. Crop rotation ___of preserving soil fertility. (A) itis one method (B) one method (C) a method is one (D) is one method 3.__ the dollar as its monetary unit in 1878. (A) Canada adopted (B) Adopted by Canada, (C) It was adopted by Canada (D) The Canadian adoption of 4.____ almost impossible to capture the beauty of the aurora borealis in photographs. (A) Being (B) His (C) There is (D) Is 5, Usually, political cartoons ____ on the editorial page of a newspaper. (A) appear (B) whose appearance (C) by appearing (D) when they appear 6.___ two major art museums, the Fogg and the Sadler. (A) Harvard University has (B) At Harvard University (C) Harvard University, with its (D) There at Harvard University 7. American director Margaret Webster ___ for her production of Shakespearean plays. (A) who became famous (B) famous as she became (C) becoming famous (D) became famous 8.__ gas tanks connected to welding equipment, one full of oxygen and the other . full of acetylene. (A) It is two (B) Of the two (C) There are two (D) Two 9. is more interested in rhythm than in melody is apparent from his compositions. (A) That Philip Glass (B) Philip Glass, who 39 (C) Philip Glass (D) Because Philip Glass 10. Compressed air___ the power to drive pneumatic tools. (A) by providing (B) provides (C) that provides (D) the provision of 11.___ by cosmic rays. (A) The earth is constantly bombarded (B) Bombarded constantly, the earth (C) Bombarding the earth constantly (D) The earth's constant bombardment 12. ___ primary colors are red, blue, and yellow. (A) There are three (B) The three (C) Three of them (D) That the three 13.___ who was elected the first woman mayor of Chicago in 1979. (A) It was Jane Byrne (B) Jane Byrne (C) That Jane Byrne (D) When Jane Byrne 14. Every computer consists of a number of systems ____ together. (A) by working (B) work (C) they work {D) that work 45. On the Moon, ___ air because the Moon's gravitational field is too weak to retain an atmosphere. (A) there is no (B) where no (C) no (D) is no 16. The Glass Mountains of northwestern Oklahoma ____ with flecks of gypsum, which shine in the sunlight. (A) they are covered (B) covered them (C) that are covered (D) are covered 17. In some cases, ___ to decide if an organism is a plant or an animal. (A) difficult if (B) it is difficult (C) the difficulty (D) is difficult 18. The first American novelist to have a major impact on world literature (A) who was James Fenimore Cooper (B) James Fenimore Cooper was (C) it was James Fenimore Cooper (D) was James Fenimore Cooper 19.___ important railroad tunnel in the United States was cut through the Hoosac Mountains in Massachusetts. (A) At first (B) It was the first (C) The first (D) As the first of 20. Generally, __ in the valleys and foothills of the Pacific Coast ranges. (A) the California poppy grown (B) the growth of the California poppy (C) the Califomia poppy grows (D) growing the Califormia poppy 21. When bats are at rest,__ hang upside-down. (A) they (B) and (C) to (D)as 22. ___ that the capital of South Carolina was moved from Charleston to Columbia. (A) In 1790 was (B) There was in 1790 (C) In 1790 (D) It was in 1790 23. Although not as important as they once were, ___a major form of transportation in North America. (A) there are still railroads (B) railroads, which are still (C) railroads are still (D) railroads still being 4 24. The Loop, which is the commercial heart of Chicago, ___ within a rectangular loop of elevated train tracks. (A) that is enclosed (B) enclosing it (C) is enclosed (D) enclosed 25.__ amino acids that serve as the basic building blocks of all proteins. (A) About twenty (B) For about twenty of (C) About twenty are (D) There are about twenty COMPLETE SENTENCES AUST ANSWER TWO QUESTIONS 3. WHO OR WHAT Is THE SENTENCE ABOUT? (rue suaut]eT> AND 2. WHAT 1S HAPPENING In THE SENTENCE? (IME PREDICATED Complete sentences are, > 42 LESSON 2 INCOMPLETE ADJECTIVE CLAUSES As mentioned in the previous lesson, there are three types of dependent clauses, all of which are tested in Structure. Adjective clauses-also called relative clauses-are the most:commonly tested of the three. You will. see one or two items involving adjective clauses on most tests. Adjective clauses are a way of joining two sentences. In the joined sentence, the adjective clause modifies (describes) a noun (called the head noun) in another clause of the sentence. It begins with an adjective clause marker. | wanted the book. The book had already been checked out. The book which | wanted had already been checked out. The adjective clause in this example begins with the marker which and modifies the head noun book. Adjective clause markers are relative pronouns such as who, that, or which or the relative adverbs when or where. ‘Adjective Clause Use Example Marker who Subject A neurologist is a doctor who specializes (people) in the “ nervous system. whom Object This is the patient whom the doctor eople) treated. whose Possessive Mr. Collins is the man whose house | (people) rented. which ‘Subject/Object _| That is a topic which interests me. (which (things) as subject) That is the topic on which | will write. (which as object of preposition) that Subject/Object | Art that is in public places can be enjoyed (people/things) | by everyone. (that as subject) The painting that Ms. Wallace bought was very expensive. (that as object) where Adverb_ Here is the'site where the bank plans to (place) build its new headquarters. when Adverb: ‘This is the hour when the children usually (time) go to bed. 43 Like all clauses, adjective clauses must have a subject and a verb. In some cases the adjective-clause marker itself is the subject; in some cases, there is another subject. The painting was very expensive. Ms. Wallace bought it. The painting which Ms. Wallace bought was very expensive. The adjective-clause marker in the joined sentence replaces it, the object of the verb bought. In the joined sentence, the adjective clause keeps the subject-Ms. Wallace-that it had in the original sentence. This is a topic. It interests me. This is a topic that interests me. The adjective-clause marker in the joined sentence replaces it, the subject of the second original sentence. In the joined sentence, the marker itself is the subject of the adjective clause. Notice that the inclusion of the pronoun it in the joined sentences above would be an error. Incorrect: *The painting which Ms. Wallace bought it was very expensive. *This is a topic which it interests me. This type of mistake is sometimes seen in distractors. When the markers which, that, and whom are used as objects in relative clauses, they can correctly be omitted. The painting Ms. Wallace bought is very expensive. (which omitted) The adjective-clause markers which and whom can also be used as objects of prepositions: That is the topic. | will write on it. That is the topic on which | will write. You may also see sentences with adjective clauses used in this pattern: quantity word + of + relative clause He met with two advisors. He had known both of them for years. He met with two advisors, both of whom he had known for years. 1 read a number of articles. Most of them were very useful. | read a number of articles, most of which were very useful. Exercise 1: Choose the best answer. 01. Cable cars are moved by cables __ underground and are powered by a stationary engine. (A) they run (B) that they run (C) run (D) that run 02. The melting point is the temperature ___ a solid changes to a liquid. (A) which (B) at which (C) which at (D) at 03. There are six types of flamingos, all___ have long legs, long necks, and beaks that curve sharply downward. (A) of them (B) that (C) of which (D) they Exercise2 : Choose the best answer. 1. Most folk songs are ballads __ have simple words and tell simple stories. (A) what (B) although (C)when (D) that 2. After its introduction in 1969, the float process ___ the world's principal method of manufacturing flat sheets of glass. (A) by which it became (B) it became (C) became (D) which became 3. In 1850, Yale University established Sheffield Scientific School, _ (A) engineers were educated there (B) where engineers were educated (C) in which were engineers educated (D) where were engineers educated 4. Many sculptures consisted of a number of large wooden structures ___in complex patterns. (A) which she arranged (B) she arranged them (C) which arranged (D) arranged them 5. Stewart E. White was a writer___ the struggle for survival on the American frontier. (A) whose novels describe (B) he describes in his novels (C) his novels describe (D) who, describing in his novels 6. Diamonds are often found in rock formations called pipes, __ the throats of extinct volcanoes. (A) in which they resemble (B) which resemble (C) there is a resemblance to (D) they resemble 7. William Samuel Johnson, ___ helped write the Constitution, became the first president of Columbia College in 1787. (A) whom he had (B) and he had (C) who had (D)had 8. Seals appear clumsy on the land, __ are able to move short distances faster than most people can run. (A) but they (B) which they (C) they (D)which 9. The instrument panel of a light airplane has at least a dozen instruments (A) the pilot must watch (B) what the pilot must watch (C) which the pilot must watch them (D) such that the pilot must watch them 10. A keystone species is a species of plants or animals __ absence has a major effect ‘on an ecological system. : (A) that its (8) its (C)whose (D) with its 46 11. The size and shape of a nail depend primarily on the function __ intended. (A) which it is — (B) for which it is (C) which it is for (D) for which is 12. In geometry, a tangent is a straight line___a curve at only one point. (A) it touches (B) whose touching (C) its touching (D) that touches 13, It was the ragtime pianist Scott Joplin __ the "Maple Leaf Rag, " perhaps the best known of all ragtime tunes. (A) wrote (B) the writer of (C) who wrote (D) writing 14, There are over 2.000 varieties of snakes, ___ are harmless to humans. (A) mostly they (B) most of them (C) most of which (D) which most 15. Smokejumpers are ___ descend into remote areas by parachute to fight forest fires. (A) firefighters (D) when firefighters (C) who, as firefighters (D) firefighters who 16. Charlotte's best known book ____she urges women to become financially independent. (A) is Women and Economics, in which (B) Women and Economics, in which (C) is Women and Economics, which (D) Women and Economics, which 47 LESSON 3 INCOMPLETE OR MISSING PARTICIPIAL PHRASES Participial phrases generally occur after nouns. They are actually reduced (shortened) relative clauses. Present participles (which always end in -ing) are used to reduce adjective clauses that contain active verbs. Minnesota, which joined the Union in 1858, became the thirty-second state. (adjective clause with active verb) Minnesota, joining the Union in 1858, became the thirty-second state. (participial phrase with a present participle) Most past participles end in -ed, but there are also many irregular forms. Past participles are used to Teduce adjective clauses with passive verbs. William and Mary College, which was founded in 1693, is the second oldest university in the United States. (adjective clause with a passive verb) William and Mary College, founded in 1693, is the second oldest university in the United States. (participial phrase with a past participle) Participial phrases can also come before the subject of a sentence. Example: Natural resources provide the raw materials to produce finished goods. (A) needed (B) are needed (C) which need (D) needing Exercise: Choose the best answer. 1. Aerodynamics is the study of the forces ___on an object as it moves through the atmosphere. (A) acting (B) act (C) are acting (D) acted 48 2. ___ for their strong fiber include flax and hemp. (A) Plants are grown (B) Plants grown (C) Plants that grow (D) To grow plants 3.____, Jose Limon's dance troupe often toured abroad. (A) The u.s. State Department sponsored it (B) Sponsored by the U.S. State Department (C) The U.S. State Department, which sponsored it (D) The sponsorship of the U.S. State Department 4. Elfreth's Alley in Philadelphia is the oldest residential street in the US, with __ from 1728. (A) houses are dated (B) the dates of the houses (C) the dating of houses (D) houses dating 5. In 1821 the city of Indianapolis, Indiana, was laid out in a design __after that of Washington, D.C. (A) pattemed (B) was patterned (C) a pattern (D) that patterned 6. ___ in front of a camera lens changes the color of the light that reaches the film. (A) Placed a filter (B) A filter is placed (C)A filter placed (D) When a filter placed 7. The Massachusetts State House, ____ in 1798, was the most distinguished building in the United States at that time. (A) completing (B) which was completed (C) was completed (D) to be completed 8. Barbara McClintock ____ for her discovery of the mobility of genetic elements. (A) known (B) who knows (C) knowing (D) is known 49. 9. The solitary scientist___ by himself has in many instances been replaced by a cooperative scientific team. (A) to make important discoveries (B) important discoveries were made (C) has made important discoveries (D) making important discoveries 10. Geometry is the branch of mathematics ___ the properties of lines, curves, shapes, and surfaces. (A) that concerned with (B) itis concerned with (C) concerned with (D) its concerns are 11,___ an average of 471 inches of rain a year, Mount Waialeale in Hawaii is the wettest spot in the world, {A) It receives (B) Receiving (C) To receive (D) Received 12. Amber is a hard, yellowish-brown ____ from the resin of pine trees that lived millions of years ago. (A) substance formed (B) to form a substance (C) substance has formed (D) forming a substance 50 LESSON 4 INCOMPLETE OR MISSING APPOSITIVES An appositive is a noun phrase that explains or rephrases another noun phrase. It usually comes after the noun which it rephrases. It may also come before the subject of a sentence. Buffalo Bill, a famous frontiersman, operated his own Wild West Show. (appositive following a noun) A famous frontiersman, Buffalo Bill operated his own Wild West Show. (appositive before the subject) Appositives are actually reduced adjective clauses. However, unlike adjective clauses, they do not contain a marker or a verb. Oak, which is one of the most durable hardwoods, is often used to make furniture. (adjective clause) Oak, one of the most durable hardwoods, is often used to make furniture. (appositive) Appositives are usually separated from the rest of the sentence by commas, but some short appositives (usually names) are not. Economist Pau! Samuelson won a Nobel Prize in 1970. In Structure items. all or part of an appositive phrase may be missing. In addition, the noun that the appositive refers to or other parts of the main clause may be missing. Example: The National Road, _ of the first highways in North America, connected the East Coast to the Ohio Valley. (A) which one (B) it was one (C) one (D) was one Exercise: Choose the best answer. 1. The Democratic Party is older than the other major American political party, (A) which the Republican party (B) the Republican party (C) it is the Republican party (D) the Republican party is 51 2. ___ relations with friends and acquaintances, playa major role in the social development of adolescents. (A) What are called peer group relations are (B) Peer group relations are (C) Peer group relations, the (D) By peer group relations, we mean 3. Joseph Henry, ____ first director of the Smithsonian Institute, was President Lincoln's advisor on scientific matters. (A) the (B) to be the (C) was the (D) as the 4. The Wassatch Range, __ extends from southeastern Idaho into northern Utah. (A) which is a part of the Rocky Mountains, (B) a part of the Rocky Mountains that (C) is a part of the Rocky Mountains (D) a part of the Rocky Mountains, it 5. __ Ruth St. Dennis tumed to Asian dances to find inspiration for her choreography. (A) twas the dancer (B) The dancer (C) That the dancer (D) The dancer was 6. The organs of taste are the __ which are mainly located on the tongue. (A) groups of cells, are taste buds (B) taste buds, are groups of cells (C) taste buds, these are groups of cells (D) taste buds, groups of cells 7. In 1878 FrederickW Taylor invented a concept called scientific management, __ of obtaining as much efficiency from workers and machines as possible. (A) itis a method (B) a method which (C) a method (D) called a method 8. A group of Shakers, ___ settled around Pleasant Hill, Kentucky, in 1805. (A) members of a strict religious sect which (B) whose members of a strict religious sect (C) members of a strict religious sect, (D) were members of a strict religious sect 52 9. In physics, __ “plasma” refers to a gas which has a nearly equal number of positively and negatively charged particles. (A) the term (B) by the term (C)is termed (D) terming 10. Norman Weiner, __ mathematician and logician, had an important role in the development of the computer. (A) who, as a (B) was a (C) whom a (D)a 11. Jerome Kern's most famoLis work is Showboat, ___ most enduring musical comedies. (A) it is one of the finest, (B) one of the finest, (C) the finest one (0) as the finest of the 12. ___ a marshland that covers over 750 square miles in North Carolina and Virginia. (A) In the Great Dismal Swamp, (B) The Great-Dismal Swamp, which (C) The Great Dismal Swamp, (D) The Great Dismal Swamp is 53 INCOMPLETE ADVERB CLAUSES A) Full Adverb Clauses An adverb clause consists of a connecting word, called an adverb clause marker (or ‘subordinate conjunction), and at least a subject and a verb. The demand for economical cars increases when gasoline becomes more expensive. In this example, the adverb clause marker when joins the adverb clause to the main clause. The adverb clause contains a subject (gasoline) and a verb (becomes). An adverb clause can precede the main clause or follow it. When the adverb clause comes first, it is separated from the main clause by a comma. When gasoline becomes more expensive, the demand for economical cars increases. The following markers are commonly seen in the Structure section: Adverb Use Example Clause Marker because cause Because the speaker was'sick, the program was. canceled. since cause ‘Since credit cards are so convenient, many people use them. although ‘opposition ‘Although he earns a good salary, he never saves (contrary any money. cause) even though | opposition Even though she was tired, she she stayed up late. (contrary cause) while contrast ‘Some people arrived in taxis while others took the subway. if condition lithe automobile had not been invented, what would people use for basic transportation? unless. condition [won't go unless you do. when time Your heart rate increases when you exercise while time ‘Some people like to listen to music while they are studying. as time ‘One train was arriving as another was departing. since time We haven't seen Professor Hill since she returned from her trip. until time Don't put off going to the dentist until you have a problem. 54 ‘once , the meeting can begin. before Before he left the country. he bought some traveler's checks. after time ‘She will give a short speech after she Is presented with the award. In Structure items, any part of a full adverb clause-the marker, the subject, the verb, and so on can be missing from the stem. B) Clause Markers with -ever Words that end with -ever are sometimes used as adverb clause markers. (In some ‘sentences, these words are actually noun-clause markers, but they are seldom used that way in Structure items.) The three -ever words that you are likely to see in the Structure Section are given in the chart below: ‘Adverb Clause Marker | Meaning Example with -ever wherever any place that... | Put that box wherever you can find room for it. wherever any time that ... They stay at that hotel whenever they're in Boston. however Anyway that ... However you solve the problem, you'll : get the same answer. C) Reduced Adverb Clauses When the subject of the main clause and the subject of the adverb clause are the same Person or thing, the adverb clause can be reduced (shortened). Reduced adverb clauses do not contain a main verb or a subject. They consist of a marker and a participle (either a present or a past participle) or a marker and an adjective. ‘When astronauts are orbiting the earth, they don't feel the force of gravity. (full adverb clause) ‘When orbiting the earth, astronauts don't feel the force of gravity. (reduced clause with present participle) Although it had been damaged, the machine was still operational. (full adverb clause) Although damaged, the machine was still operational. (reduced ‘clause with a past participle) 55 Although he was nervous, he gave a wonderful speech. (full adverb clause) Although nervous, he gave a wonderful speech. (reduced clause with an adjective) You will most often see reduced adverb clauses with the markers although, while, if, when, before, after, and until. Reduced adverb clauses are NEVER used after because. D) Prepositional Phrases with the Same Meaning as Adverb Clauses There are also certain prepositions that have essentially the same meaning as adverb- clause markers but are used before noun phrases or pronouns, not with clauses. Preposition Related Example Marker because of because/since | He chose that university because of its fine reputation. due to because/since | The accident was due to mechanical failure. on account of | because/since | Visibility is poor today on account of air pollution. in spite of although/even | He enjoys motorcycle riding in spite of the though danger. despite although/even | Despite its loss, the team is still in first place. though during when/while Her father lived in England during the war. In structure items where the correct answer is an adverb-clause marker, one of these words often appears as a distract or. Example: 01. No one knows what color dinosaurs were ___ no sample of their skin has survived. (A) because of (B) because that (C) itis because (D) because 02. __ rises to the surface of the earth, a volcano is formed. (A) Liquid magma (B) Whenever liquid magma (C) Liquid magma, which (D) That liquid magma 03.___ invisible to the unaided eye, ultraviolet light can be detected in a number of ways. (A) Although is (B) Despite (C) Even though it (D) Although 04. Because ___, alabaster can be easily carved. (A) is soft (B) softness (C) of its softness (D) of soft Exercise: Choose the best answer. 1. Small sailboats can easily capsize ___ they are not handled carefully. (A) but (B) which (C)if (D)so 2. they are tropical birds, parrots can live in temperate or even cold climates. (A) Despite (B) Eventhough (C) Nevertheless (D) But 3. _ added to a liquid, antifreeze lowers the freezing temperature of that liquid. (A) That (B) Asis (C)Itis (D) When 4. ___advertising is so widespread in the United States, it has had an enormous effect on American life. (A) Why (B)The reason (C) On account of ‘(D) Since 5. ___ towards shore, its shape is changed by its collision with the shallow sea bottom. (A) During a wave rolls (B) As a wave rolls (C) A wave rolls (D) Awave's rolling 6. ____ are increasingly linked over long distances by electronic communications, but many of them still prefer face-to-face encounters. (A) Although people (B) Despite people (C) Today people (D) The fact that people 7. ___ together in one place, they form a community. (A) When people who live (B) When people living (C) Whenever people live (D) Whenever living people 8. aa managed by an independent governor and board of directors, the Bank of Canada is owned by the Canadian government. (A) And yet {B) In spite of it (C) Although (D) It is 57 9. ___ pieces of rope are of different thickness, the weaver's knot can be used to join them. (A) Two of (B) What two (C) Two such (D) If two 10.___, the seeds of the Kentucky coffee plant are poisonous. (A) Until they have been cooked (B) Cooking them (C) They have been cooked {D) Cooked until 11. Natural silk is still highly prized _-_ similar artificial fabrics. (A) although is available (B) despite there are available (C) in spite of the availability of (D) even though an availability of 12. Cattle ranches are found almost ___ in Utah. (A) wherever (B) everywhere (C) overall (D) somewhere 13.___ through a prism, a beam of white light breaks into all the colors of the rainbow. (A) When shines (B) It shines (C) It is shone (D) When shone 14. ___ most people think of freezing as a relatively modem method of food preservation, it is actually one of the oldest. (A) Even (B) As though (C) However (D) Even though 15.___ large bodies of water never freeze solid is that the sheet of ice on the surface protects the water below it from the cold air. (A) Because (B) Why do (C) The reason that (D) For the reason 58 16.___ granted by the Patent Office, it becomes the inventor's property and he or she can keep it, sell it, or license it to someone else. (A) Once a patent is (B) When a patent (C)A patent, once (D) A patent, whenever it 17. Owls can hunt in total darkness ___ their remarkably keen sense of smell. (A) since i (B) because of (C) the result (D) that 18.___ most bamboo blooms every year, there are some species that flower only two or three times a century. (A) Whenever (B) That (C) White (D) However 59 LESSON 5 INCOMPLETE NOUN CLAUSES Noun clauses are the third type of subordinate clause. They begin with noun-clause markers. Noun clauses that are formed from statements begin with the noun-clause marker that. Noun clauses formed from yes/no questions begin with the noun-clause markers. whether or if. Those formed from information questions begin with wh- words: what, where, when, and so on. Dr. Hopkins’ office is in this building. (statement) I'm sure that Dr. Hopkins’ office is in this building. Is Dr. Hopkins' office on this floor? (yes/no question) | don't know if (whether) Dr. Hopkins' office is on this floor. Where is Dr. Hopkins’ office? (information question) Please tell me where Dr. Hopkins’ office is. Notice that the word order in direct questions is not the same as it is in noun clauses. The noun clause follows statement word order (subject + verb), not question word order (auxiliary + subject + main verb). Often one of the distractors for noun-clause items will incorrectly follow question word order. *1 don't know what is her name. (Incorrect use of question word order) I don't know what her name is. (Correct word order) *She called him to ask what time did his party start. (Incorrect use of question word order) She called him to ask what time his party started. (Correct word order) Noun clauses function exactly as nouns do: as subjects, as direct objects, or as complements after the verb fo be. When the meeting will be held has not been decided. (noun clause as subject) The weather announcer said that there will be thunderstorms. (noun clause as direct object) 60 This is what you need. (noun clause after to be) Notice that when the noun clause is the subject of a sentence, the verb in the main clause does not have a noun or pronoun subject. In Structure items, the noun-clause marker, along with any other part of the noun clausesubject, verb, and so on-may be missing from the stem, or the whole noun clause may be missing. Example: 01. ____ was caused by breathing impure air was once a common belief. (A) Malaria (B) That malaria (C) Why malaria (D) Because malaria 02. One basic question psychologists have tried to answer is (A) people learn (B) how do people leam (C) people leam how (D) how people learn Exercise: Choose the best answer. 1.____ begin their existence as ice crystals over most of the earth seems likely. (A) Raindrops (B) If raindrops (C) What if raindrops : (D) That raindrops 2. Scientists cannot agree on ____ related to other orders of insects. (A) that fleas are (B) how fleas are (C) how are fleas (D) fleas that are 3. It was in 1875 ___ joined the staff ofthe astronomical observatory at that university. (A) that Anna Winlock (B) Anna Winlock, who (C) as Anna Winlock (D) Anna Winlock then 61 4. __ is anarrow strip of woods along a stream in an open grassland. (A) Ecologists use the term "gallery forest" (B) What do ecologists call a "gallery forest" (C) "Gallery forest” is the term ecologists use (D) What ecologists call a “gallery forest” 5. ____ developed very rapidly in Alabama primarily because of its rich natural resources. (A) That heavy industry (B) Heavy industry (C) Heavy industry that was (D) When heavy industry 6. ___ so incredible is that these insects successfully migrate to places that they have never even seen. (A) That makes the monarch butterflies’ migration (B) The migration of the monach butterflies is (C) Wnat makes the monarch butterflies’ migration (D) The migration of the monarch butterflies, which is 7. Art critics do not all agree on what ____a painting great. (A) qualities make (B) are the qualities for making (C) qualities to make (D) do the qualities that make 8. In order to grow vegetables properly, gardeners must know __ (A) what the requirements for each vegetable are (B) that the requirements for each vegetable (C) what are each vegetable's requirements (D) that is required by each vegetable 9. When ____ is not known. (A) was the wheel invented (B) the invention of the wheel (C) inventing the wheel (D) the wheel was invented 10. For many years people have wondered ____ exists elsewhere in the universe. (A) that life (B) life which (C) whether life (D) life as it 62 11.__ of all modern domestic poultry is the red jungle fowl is widely believed. (A) The ancestor (B) The ancestor is (C) How the ancestor (D) That the ancestor 12. ___ the right side of a person's brain is dominant, that person is left-handed. (A) That (B) If (C) Which (D) For 63 LESSON 6 MISSING OR INCOMPLETE PREPOSITIONAL PHRASES A prepositional phrase consists of a preposition (in, at, with, fm; until. and so on) followed by a noun phrase or pronoun. which is called the prepositional object. Prepositional phrases often describe relationships of time and tocation, among others. In the auturnn, maple leaves tum red. Beacon Hill is one of the most famous neighborhoods in Boston. ‘With luck, there won't be any more problems. This house was built by John's grandfather. Often, prepositional phrases come at the beginning of sentences, but they may appear in other parts of the sentence as well. The correct answer for this type of item may be a preposition, its object, or both, as well as other parts of the sentence. You may see prepositions in distractors, especially before the subject of a sentence. Remember, the object of a preposition cannot correctly be the subject of a sentence, as in these examples: ‘in the autumn is my favorite season. “Without a pencil is no way to come to a test. Example: 01.____ the unaided eye can see about 6,000 stars. (A) A clear night (B) It's a clear night (C) On a clear night (D) When a clear night 02.___ all the field crops grown in the United States are harvested with machines called combines. : (A) Of nearly (B) Nearly (C) That nearly (D) Nearly of Exercise: Choose the best answer. 1. Dynamite is ordinarily detonated ___ called a blasting cap. (A) a device is used (B) that a device (C) with a device (D) the use of a device 2.____ seed of a flowering plant is covered by a dense protective coat. (A) On each (B) Each (C) Each of (D) That each 3.____ 1900 there were some 300 bicycle factories in the United States, and they produced over a million bicycles. (A) In (B) Because in (C) It was in (D) That in 4. A thick layer of fat called blubber keeps whales warm even ____ coldest water. (A) although the (B) inthe (C) the (0) of the 5.____ the United States, the general movement of air masses is from west to east. (A) Across (B) To cross (C) They cross (D) It's across 6. The bark of a tree thickens ___ (A) with age (B) it gets older (C) as older (0) by age 7. A substance that is harmless to a person who has no allergies can cause mild to serious reactions ina person __ allergies. (A) has (B) which having (C) can have (D) with 65 8. In 1886 a number of national unions formed the American Federation of Labor ___- (A) Samuel Gompers was its leader (B) under the leadership of Samuel Gompers (C) which, under Samuel Gompers' leadership (D) Samuel Gompers led it 9. Harmonicas, autoharps, and kazoos ____ folk instruments. (A) are examples. (B) for example (C) are examples of (D) as examples of 10.____ charming shops and restaurants, Old Town is the most picturesque section of Albuquerque. (A) With its (B) Its (C) Because its (D) For its 41, ___, such as banking and travel, in which computers are not a convenience but a necessity. (A) Where some industries (B) In some industries (C) Some industries (D) There are some industries 42. One of the oldest large suspension bridges still__ today is the George Washington Bridge between New york City and Fort Lee, New Jersey. (A) uses (B) is used (C) the use of (D) in use 66 PRACTICE TEST Directions: The following sentences are incomplete. Beneath each of these sentences, there are four words or phrases, marked (A), (B), (C), and (D). Choose the one word or phrase that best completes the sentence. 1. __, an organism must be able to adapt to changing factors in its environment. (A) If survival (B) For surviving (C) To survive (D) It survives 2. The art of landscape architecture is ___ that of architecture itself. (A) almost as old as (B) as almost old (C) almost as old than (D) old as almost 3. The Mummers’ Parade has __ every year on New Year's Day since 1901. (A) holding é (B) been holding (C) held (D) been held 4. Rarely more than ftfty miles from the coast. (A) redwood trees grow (B) redwood trees do grow (C) grow redwood trees (D) do redwood trees grow 5. Microorganisms live in extreme conditions of heat and cold where __ other organisms can survive. (A) not (B) never (C)no (D) none 6. The higher one rises in the atmosphere, the temperature generally becomes. (A) colder than (B) the colder (C) the colder as (D)is colder 7. Medical researchers are constantly looking for ways to control, _, and cure diseases. (A) prevention (B) preventing (C) prevent (D) to prevent 67 8. Nerve cells, or neurons, ___in the human body. (A) the most complex cells are (B) are the most complex cells (C) most complex the cells are (D) most are the complex cells 9. Released in 1915, __ (A) D. W Griffith made an epic film about the Civil War, Birth of a Nation (B) the Civil War was the subject of D. W Griffith's epic film, Birth of a Nation (C) D. W Griffith's epic film Birth of a Nation was about the Civil War (D) the subject of D. W Griffith's epic film Birth of a Nation was the Civil War 10,___ on barren slopes can help prevent erosion. (A) Planting trees (C) In order to plant trees (8) For trees to be planted (D) Trees are planted 44. Vermont is the only state in New England ____an Atlantic coastline. (A) without (B) not with (C) which not having (D) doesn't have 12. In 1867, Hiram R. Revels ___ the first black to be elected to the US Senate. (A) becoming (B) became (C) to have become (D) has become 413. Jupiter's moons can be easily seen through ___ binoculars or a small telescope . (A) either (8) if (C) whether (D) or 14. The Colorado beetle is a beautiful insect, __ it causes a great deal of damage to food crops. (A) but (B) what (C) or (D) that 15. Judge Francis Hopkins is probably best known as a signer of the Declaration of Independence, but he also excelled as a poet, _, and an orator. (A) as a musician (B) by playing music (C) a musician (D) he played music 46. It is said he was aman, ____ to have the vision of an eagle and courage of a lion. (A) who appeared (B) he appeared (C) that appears (D) and appears 17. Before Johnson & Smith reached great heights in the business world, __ encountered many great difficulties in promoting their theories and methods. A. they B. who C. which D. and 18. After the discovery of the abandoned getaway vehicle, _ believed to be hiding in the nearby Riverside forest region. A. that the bank robber is B. the bank robber who C. the bank robber is D. the bank robber who is 19. A gifted scientist, Newton ___ some of the most fundamental laws in the history of science. A. keeps discovering B. who discovered C. the discoverer of D. discovered 20. George Washington once said that ___ have virtue enough to withstand the highest bidder. A. few men B. the few men C. few are the men D. the men are few 21. ___ getting the highest result in the class, John still had problems with the teacher. (A) Despite of (B) In spite of (C) Even though (D) Nonetheless 22. __airis composed of about 78 percent nitrogen and only about 21 percent oxygen, is a little known fact on the streets. A. How that B. That C. When D. However 23.___ he was seen to be an aggressive politician, he was a quiet and loving family man at home. A. Although B. Despite C. In spite of D. Nevertheless 69 24. __ the variable drops by a unit of 1, the rank drops by X amount. A. Why B. Whenever C. How D. What 25. This method is widely used __ algorithm is not only effective but also very simple. A. because its is because D. because of its 26. This new service will be available to all users ___ up for paid membership. A. that signed B. that signed it C. which signed D. sign 27. That'is a story of hardship __ our own situation into perspective. A. puts: B. it puts C. that it puts D. that puts: 28. John Smith, _ of economic crimes, tax evasion and fraud, is being accused of attempted murder now. A. of that he was accused B. that was accused C. whom he was accused D. who was accused 29. The process uses an innovative digital technology __. the products with as many colors as the image contains. (A) imprints (B) that imprints it (C) that imprints (D) that it imprints 30. The police were-greatly outnumbered by rioters, __ran into the hundreds. A. whose figures B. those figures C. that its figures D. its figures that 70 31. -__ explores the nature of guilt and responsibility and builds to a remarkable conclusion. A. The written beautifully novel B. The beautifully written novel C. The novel beautifully written D. The written novel beautifully 32. Over time the young students will perfect the art of piano playing. After all, such __ needs delicate handling. A. a tuned instrument finely B. an instrument tuned finely C. a finely instrument tuned D. a finely tuned instrument 33. Honore de Balzac said "The errors.of ____, from their faith in the good, or their confidence in the true". A. women spring, almost always B. almost always, women spring C. almost women, always spring D, almost spring, always women 34. In that particular department of the company, production __ day and night. (A) going often keeps (B) keeps often going (C) keeps going often (D) often keeps going 35. The tenants were asked to throw all recyclable trash into__. A. the green big plastic bag B. the big plastic green bag C. the big green plastic bag D. the green plastic big bag 36. The causes of gamma-ray bursts throughout the universe, and how the gamma rays are actually produced, _until recently remained a mystery. A. have B. have been C. has been D. has 37. That the legal drinking age _ lowered is a hot topic for debate in many states. A. should have B. which should C. should be D. should 71 38. Discovery of these ancient anthropic markings __ our understanding of how these early humans interacted with their environment. A. has broadened B. have broadened C. broaden D. will have broadened 39. The rivalry between the two communities stems from ancient times and openly __ to this day. A. persist B. which persist C. persists D. which persists 40. Despite the'simplicity of their construction, the ancient systems __ exhibit very complicated behavior. A. find to B. found to C. are found to D. were found to 41. In general, ____ have a professional obligation to protect confidential sources of information. ‘A. which joumalists B. journalists C. journalists, they D. journalists that 42.___ of Elvis Presley is banned in seven different countries across the world. A. The music is B. That the music C. The music which D. The music 43. For____ you who don't know, “Rainbow’ was a credible children's TV show from the 70's and 80's. A. those of B. those in C. these of D. these in 44. As was the case throughout the United States, __ was subject to higher gas prices during the winter of 2002-2003. A. for New York C. New York which B. for New York as well D. New York 72 45. ___ in history caused as much shock and grief worldwide as the 2004 tsunami disaster in Asia. A. None natural disaster B. That natural disaster C. No natural disaster D. The natural disaster 46. Surprisingly cost was regarded __—_ important factor in choosing a new cell phone by the three teenagers. Aas the least B of the least Cin the least D to the least 47. Maine's coastline is a major attraction and a vista of sandy beaches contrasted __ rockbound shoreline. Ato the rugged B by the rugged C on the rugged Dat the rugged 48. John knows that he had better __his algebra skills before the mid-term exam. A. brush up on B. brush on up C. brushing up on D. brushing on up 49. The pressure __._ was intense after his team lost five basketball games in a row. A. under the coach B. over the coach C. of the coach D. on the coach 50. In ___, the team has begun to show some form again and has won some big games. ‘ (A) few weeks (B) few past weeks: (C) the past few weeks (D) a few weeks since 73 READING SECTION The Reading test section measures your ability to understand academic passages in English. You will read one passage and answer questions about it. To do well on this test, you should therefore work in these areas to improve your score: 1. You must work on improving your knowledge of the English language skill. 2. You must understand the test-taking strategies specific to the version of the test that you are taking. 3. You must take practice tests wih a focus on| LOVE TO READ applying the appropriate language skis and test- Beigars rina taking strategies. Tips: 1. Be familiar with the directions. 2. Do not spend too much time reading the passage. You do not have time to read each passage in depth. It is quite possible to answer the questions correctly without first reading the passage thoroughly. 3. Do not worry if a reading passage is on a topic you are unfamiliar with. All of the information that you need to answer the questions is included in the passages. You do not need any background knowledge to answer the questions. 4. Do not spend too much time on a question you are unsure of. If you do not know the answer to a question, simply guess and go on. You can retum to this question later in the section if you have time. 5. Guess to complete the section before time is up. There is no penalty for guessing, so it can only increase your score to guess the answers to questions that you do not have time to complete. 74 LESSON 1 Topic and Stated Main Idea Multiple-Choice Exercise Review: The topic is a word, name, or phrase that answers the questions, "Who or what is this paragraph about?" The stated main idea answers the question, "What is the author's most important point about the topic?" It is always a sentence, and it must contain the topic. Directions: Read each paragraph and determine its topic. Then select the answer choice that tells the stated main idea sentence. Passage One Passing fears are common in early childhood. Many 2- to 4-year-olds are afraid of animals, especially dogs. By 6 years, children are more likely to be afraid of the dark. Other common fears are of thunderstorms, doctors, and imaginary creatures. The topic of this paragraph is A. fear. i B. passing fears. C. two-to-four year olds fear of animals. D. common fears. The stated main idea of this paragraph is ‘A. Passing fears are common in early childhood. B. ‘Many 2- to 4-year olds are afraid of animals, especially dogs. ©. By 6 years, children are more likely to be afraid of the dark. D. Other common fears are of thunderstorms, doctors, and imaginary creatures. Passage Two ‘Congratulations! You have been offered a job. The time has come to negotiate your salary. The question to ask yourself is: "How much am | worth?" Your answer will likely affect the outcome of your salary negotiation. The point is, in order to negotiate the best possible salary, you must convince both yourself and the employer of the value that you will bring to the job. Hi topic of this paragraph is congratulations. B being offered a job. C. asking yourself how much you are worth. D. negotiating the best possible salary. ve Stated main idea of this paragraph is You have been offered a job. The time has come to negotiate your salary. . The question to ask yourself is: “How much am | worth?" The point is, in order to negotiate the best possible salary, you must convince both yourself and the employer of the value that you will bring to the job. pop> 75 Passage Three There are gender differences in adolescents’ satisfaction with their bodies. Compared with boys, girls are usually less happy with their bodies and have more negative body images. Also, as puberty proceeds, girls often become even more dissatisfied with their bodies. This is probably because their body fat increases. In contrast, boys become more satisfied as they move through puberty, probably because their muscle mass increases. The topic of this paragraph is A. adolescents’ satisfaction with their bodies. B. girls' negative body images. C. puberty. D. boys’ muscle mass during puberty. The stated main idea of this paragraph is A. There are gender differences in adolescents' satisfaction with their bodies. B. Compared with boys, girls are usually less happy with their bodies and have more negative body images. C. Also, as puberty proceeds, girls often become even more dissatisfied with their bodies. D. In contrast, boys become more satisfied as they move through puberty, probably because their muscle mass increases. Passage Four One of the big programming surprises of the 2002 summer TV series was a show on the Fox Network called American Idol, a talent search that highlighted several aspiring performers. The final episode of the show gamered the biggest audience share among 18- to.49-year olds that the network has ever had. Based on a British series called Pop /dol, American Idol is another in a long list of shows that the United States has imported. In fact, many popular U.S. TV shows originated overseas. These include the quiz shows Who Wants to Be A Millionaire and The Weakest Link, both also from Britain . Other examples include Survivor, imported from Sweden; Big Brother, based on a Dutch series of the same name; and TLC's Trading Spaces, based on the British series, Changing Rooms. The topic of this paragraph is A. big programming surprises. B. American Idol. C. popular U.S. TV shows. D. British quiz shows. The stated main idea of this paragraph is A. One of the big programming surprises of the 2002 summer TV series was a show on the Fox Network called American Idol, a talent search that highlighted several aspiring performers. B. Based on a British series called Pop Idol, American Idol is another in a tong list of shows that the United States has imported. C. In fact, many U.S. TV shows have first originated overseas. D. These include the quiz shows Who Wants to Be A Millionaire and The Weakest Link, both also from. Britain. 76 Passage Five Don't wait for your company to send you to school. Determine your needs and ask about the company’s training program. If they don't have one, sign up for classes at a local college. When it comes to your profession, you should be a lifelong learner. Put a high priority on learning new skills and on personal growth and professional development. Learn new software technology and improve interpersonal and writing skills. The topic of this paragraph is A. waiting for your company to send you to school. B. the company's training program. C. being a lifelong learner. D. new software technology. The stated main idea of this paragraph is A. Don't wait for your company to send you to school. B. Determine your needs and ask about the company's training program. C. When it comes to your profession, you should be a lifelong leamer. D. Learn new software technology and improve interpersonal and writing skills. Review 1: The main idea must always be expressed as a sentence. It must also contain a topic and must make complete sense by itself. Here are some examples to illustrate the difference between a topic and a main idea sentence: Reality TV shows. No. It's a phrase, so it could be a topic. It's not a sentence, however, so it couldn't be a main idea. In recent years, they have become very popular. No. It's a sentence, but it does not make complete sense by itself. It doesn't tell what “they” refers to. Therefore, it could not be a stated main idea. In recent years, reality TV shows have become very popular. Yes. This could be a stated main idea because the sentence makes complete sense by itself. 77 EXERCISES Directions: Select TRUE for any item that could be used as a main idea sentence. Do not base your decision on whether an item has a period at the end of it. There are periods after all of the items, but not all of them would work as main idea sentences. Examples: 04. 02. 03. 05. 06. 07. 08. 09. 10. 11. 13. 14. 15. 16. Bra 18. 19. 20. TRUE Online courses are very convenient. Yes, because it is a sentence and makes sense by itself. Why video games are so popular. He is the top-ranked golfer in the world. In college and on the job, it is important to be able to follow written directions. Itis a group of related sentences. ‘Students learn in different ways. Previewing a chapter is the first step in reading the assignment. Itis the best way of preventing the common cold. ‘There are three main types Instant messaging is a popular way to communicate. They can be dangerous to children and pets. Most of them do not work: A mnemonic device is used to aid in remembering. Whether the new schedule will work. Every hour 70 new blogs are created. The public should vote against this plan. How working too many hours affects college students’ success. Working too many hours affects college students' academic success. Sleep well. ‘There are differences in male and female brains. She could run as fast as a racing car. 78 Review 2: A main idea is always a complete sentence that states an important point about the topic. That's why you can often find the topic in the main idea sentence. Main ideas are never questions; they're always statements. Directions: Mark which of the following statements would be considered stated main ideas. 01. There are several benefits to having a college degree. 02. Ways to save money for college. 03. How to get financial assistance in college 04. Finding money to pay for college can be a challenge. 05. What are the best ways to save for college? Authors use different styles to convey main ideas. 1. Some main ideas will introduce the major points that the paragraph will discuss. There are several types of marketing methods that would be appropriate for this product. (The several types would be the major details.) Three main factors are important to consider when deciding upon the correct way to solve a crime. (The three factors would be the major details.) 2. Some main ideas are a summary of the paragraph. Mozart was one of the most influential composers of all time. (The major details would explain why he was the most influential composer.) Thomas Edison worked tirelessly to create new inventions and processes. (The major details would explain how he worked to create new inventions and processes.) 3. In definition pattem paragraphs, the definition of the term is the main idea. It is usually followed by an explanation and may include examples in the supporting details. Immediate memory is the temporary memory where information is processed briefly (in‘seconds) and subconsciously, and then is either blocked or passed on to working memory. Self-concept is the perception of who we are and how we fit into the world. 79 4. Other main ideas may be a conclusion of a study, experiment, or discussion. The results of the study show that, over the period of a lifetime, students who do not get a college degree will earn less than half the income of those who do. Directions: Read the following paragraph. Find the topic by underlining the subject that is most often mentioned. Then, decide which sentence states the main point that the author is trying to make about the topic. This will be the topic sentence. When you think you have found it, ask yourself, “Do most of the other sentences tell me more about this idea?” If the answer is yes, you have the correct topic sentence. When trying to save money, you need to know about your options. Most people save their money in a savings account at a bank. They are paid interest from the bank for allowing the bank to hold and use their money. If you save money in a money market account, your interest rate will be determined by how well the stock market is doing. Although you won't lose any of your original investment, you won't be making much interest when interest rates are low. Another way to save money is to buy a certificate of deposit, or CD. You cannot cash in the certificate until it reaches a certain age, but CDs are a good investment if you don't need the money right away. 01. What is the topic of this passage? A.- savings accounts C. banking B. money market accounts D. ways to save money 02. What is the topic sentence? A. Most people save their money in a savings account at a bank. B. When trying to save money, you need to know about your options. C. If you save money in a money market account, your interest rate will be determined by how well the stock market is doing. D. Saving money today will keep you from financial trouble in the future. 80 LESSON 2 IMPLIED MAIN IDEA Have you ever looked everywhere for something, only to find it sitting right in front of you the whole time? Searching endlessly for something you know is there can be extremely frustrating. This is also true in a situation where you're trying to figure out what someone is talking about. If we have to really try to figure out what people are talking about, it probably means they're using an implied main idea, or the primary point of a sentence or passage that is not clearly stated, but instead deduced from surrounding details. Of course, the main idea of a sentence or passage is its primary point or argument. And in everyday conversation, we tend to state our main ideas clearly, not wanting anyone to be distracted or confused. For instance, this lesson's primary point is to talk about implied main ideas, and ‘everywhere you look here, you should be able to find clear evidence of that fact. We might digress here and there to discuss particular details, but it's always evident that we're talking about implied main ideas. But how do we find implied main ideas when they're not the center of attention? Finding an Implied Main Idea Why would we ever need to imply a main idea when it's so easy to say it outright? People use implied main ideas for all sorts of reasons - from disguising their real intentions, to keeping us on our toes. In fact, you've probably even used an implied main idea or two in the form of what we might call ‘dropping hints." Take for example this seemingly rambling account from a theoretical 13-year-old. Did you know snowboarding started from surfing? They even have snowboarding in the Olympics now! | wonder how long it takes to build a snowboard? | saw one the other day, though, down at the mall... Although it might look like these sentences are only thinly connected, their implied main idea actually keeps them all tightly linked. And here's how we can tell... Read the passage entirely first. All the details might not make sense immediately, but you should start to see patterns. For instance, the young teenager appears to be jumping from one idea to the next; but if we look closely, we can see that snowboards feature prominently throughout the passage. Examine individual details to see how they relate to the common thread. The first sentence relates snowboarding to surfing (maybe the parents are surfers?); the second hypes its place in the Olympic Games; while the third and last imply the difficulties of building a board, and the efficiency of finding one at the mall. 81 Put the details together to find out what the main idea is. We can look at the details together to see that the implied main idea of this teenager's hint-dropping is that he or she wants a snowboard: each 4 detail attempts to add some sort of value to the sport so that the parents will finally be convinced to buy the snowboard at the mall. Re-reading the passage or summarizing it can also be helpful at this stage. in the process. W Ys Now that we've seen how to find the implied main idea hidden in plain sight, let's take a look at a couple more examples. EXERCISE 1 Directions: The topic is given for each of these paragraphs. Remember that the topic must be part of the main idea sentence. Read each paragraph. Then select the answer choice that expresses its implied main idea. Topic: leadership styles Autocratic leaders are hands-on leaders who keep strict contro! over group members and their activities. They ask few questions, make the decisions, give orders, and are likely to use coercion to make others carry out their assignments. Laissez-faire leaders are hands-off leaders who leave most of the decisions to the group and tend not to get involved. Democratic leaders encourage group participation in decision-making and problem solving. Their style falls between the other two extreme. A. Democratic leaders are more effective than autocratic leaders. B. There are three leadership styles. C. Autocratic leaders are strict, laissez-faire leaders are hands-off leaders, and democratic leaders encourage group participation in decision-making. D. Employees prefer to work with democratic leaders. Topic: Americans waiting to marry (age at which Americans marry) Are Americans today waiting longer to get married? According to 2003 Census Bureau figures, the answer is yes. The Associated Press reports that one-third of men are still single when they reach age 34 and that nearly one-quarter of women are still single at that age. Compared with data for 1970, these figures are four times higher. In 1970, the percent of never-married men aged 30-34 was 9 percent; the tate has risen to 33 percent. The percent of never-married women increased from 6 percent to 23 percent. The typical marriage age for men in 2003 was 27.1 years, up from 25,3 in 1970. The typical age for women rose from 20.8 to 23.2. A. Are Americans today waiting longer to get married? B. Men marry at a later age than women do. C. Americans today are waiting longer to get married. D. More men than women are still single at age 34 82 Topic: results of a survey about teens and money A recent survey revealed some. shocking results. One in five teens does not know that if you take out a loan, you must pay interest in addition to repaying the loan. One teen in four has the mistaken notion that financial aid will take care of all their college expenses. And one teen in three thinks that Social Security payments will provide all the money they need when they retire. : A. Arecent survey revealed some shocking results: one in five teens does not know that if you take out a loan, you must pay interest in addition to repaying the loan. B. Teenagers do not know. anything about money matters. C. Arecent survey revealed some shocking results about how little understanding teens have about money matters. D. One teen in four has the mistaken notion that financial aid will take care of all their college expenses. And one teen in three thinks that Social Security payments will provide all the money they need when they retire. Topic: fiction (definition of) Novels and short stories are types of fiction. Drama is another example. Fairy tales and fables are also fiction. It is a type of narrative writing that comes from the. imagination of the author rather than from history or fact. A. There are many types of literature. B. Novels, short stories, drama, fairy tales and fables are types of fiction. C. Fiction is a type of narrative writing that comes from the imagination of the author rather than from history or fact. D. Novels and short stories are types of fiction. Topic: how far it is to the Sun (distance to the sun) How far is it to the Sun? It's so far that it's hard to comprehend. In actual distance, it's approximately 93 million mites. The distance changes slightly as the Earth travels around the Sun. Suppose it were possible to take a jetliner there. Traveling at a little over 550 mph, it would take nearly 20 years to get there. Even if you could travel at 25,000 mph, it would take five months.to reach the Sun. A. How far is it to the sun? B. It's so far to the Sun that it's hard to comprehend. C. In actual distance, i's approximately 93 million miles to the Sun. D. It takes a long time to get to the Sun, no matter how you travel. 83 EXERCISE 2 Directions: Read each paragraph and determine its topic. Then select the answer choice ‘that expresses its implied main idea. (Remember that the topic must be part of the main idea sentence.) Passage One How do migrating animals find their destination? They navigate in a variety of ways. Fish use their sense of smell to recognize their migration paths and are guided by changing water temperatures. Birds use the position of the sun to orient themselves. Some birds have magnetic particles in their ear mechanisms that act as a compass. Mammals rely on their memory. Some elephant trails have been used for hundreds of years. The topic of this paragraph is: A. destinations. B. fishes’ sense of smell. ° C. how birds orient themselves. D. migrating animals The implied main idea is: A. How do migrating animals find their way? B. Migrating animals navigate in a variety of ways. C. Mammals rely on their memory. D. Fish and birds have special methods for finding their way. Passage Two Psychologists say that a sigh can express satisfaction and contentment. Frequent sighing can’ suggest despair and depression. It can also express disgust or frustration. It can express several emotions, but there is no one correct way to interpret its meaning. The topic of this paragraph is: A. psychologists. B. a sigh; sighing. C. despair and depression. D. interpreting emotions. He implied main idea is: A. Psychologists say that a sigh can express satisfaction and contentment, and frequent sighing can suggest despair and depression. a aoe can express several emotions. Assign can express emotions, such as disgust or frustration. . A sigh can express several emotions; but there is no one correct way to interpret its meaning. poe LESSON 3 MAKING INFERENCING What is an inference? It is a logical guess based on details and clues in the text. It is like a hypothesis in a science project. In order to make good inferences, we need to pay attention to the details the writer gives us about what they know, and then connect those details to the plot. It is often necessary to read between the lines because writer cannot include all the possible information about a topic or situation. Writers leave out information that they think readers will know already or will be able to guess. Observe the picture on the right. What is happening? What inference can you make? In reading a text, you have to be able to distinguish between facts and inferences. An author usually presents facts about a situation or topic and also makes inference from the facts. Facts are statements of information that can be verified while inferences are educated guesses that are based on facts. For example: (Fact) On April 5, 2017, residents of Jakarta elected their new governor and vice governor with 52.8% of their votes based on quick counts. (Inference) People of Jakarta want changes in their society and economy. 85 EXERCISE 1 A. Directions: Read the following passage and answer the questions that follow. Unlike parts of Indonesia closer to the Asian mainland, Flores has been an island for at least a million years. As is the case with islands elsewhere, its fauna is evolved in its own way, producing creatures larger or smaller than their mainland relatives: a lost world of tiny elephants, giant rats, Komodo dragons and even larger extinct lizards. The isolation had its effects on the human inhabitants. One of the most surprising things about the Liang Bua skeleton is its size: in life, no: more than a meter (about 3 ft) tall, about the same size as one of the giant rats. Living in a hole in the ground and chased by lizards of giant proportions, the creature has been nicknamed “hobbit” by some researchers-a reference to the small, hole-dwelling heroes of The Lord of the Rings. For Brown, it was the smaliness of the skull which showed that. Homo floresiensis was truly different. When he measured the skull volume and found it a chimp-sized 380 cc, he says his jaw “dropped to my knees. Small stature is easy to explain but small brain capacity is a bigger problem-t still is.” Yet these tiny-brained creatures were skilled enough to make finely crafted stone tools. (Source: Advanced Reading Power, BS Mikulecky and L. Jeffries, 2007) 01: From the facts here, what can you infer about the process of evolution on islands? 02. “This isolation had its effects on the human inhabitants.” Is this a fact or an inference? 03, What do scientists usually infer about small brain size? 04. Why was Brown so surprised about the small brain size? B. Directions: Read the following passage and answer the question. One of the rarest animals in the world is the okapi. They live in the rain forest in Central Africa. Few people have seen one alive. Every few years someone claims to see one, but they never have proof. A few years ago, some scientists who were studying plants found a skeleton, teeth and some skin in a remote area of the rain forest. They were excited, as they believed they had found the remains of a rare okapi. They drew a picture of what they thought the animal might have looked like when it was alive. The scientists then sent the picture along with the skeleton, teeth and skin to a scientific institution in Dallas, Texas. What did the scientists probably do with the skeleton, teeth and bones? A Sent them to a place with more experienced scientists. B Kept them for souvenirs. C Studied them carefully and compared them to other animals D Sold them to museums that displayed them 86 C. Directions: Read the following passage and answer the question. One of the rarest animals in the world is the-okapi. They live in the rain forest: in Central Africa. Few people have seen one alive. Every few years someone claims to see one, but they never have proof. A few years ago, some scientists who were ‘studying plants found a skeleton, teeth and some skin in a remote area of the rain forest. They were excited, as they believed they had found the remains of a rare okapi. They drew a picture of what they thought the animal might have looked like when it was alive. The scientists then sent the picture along with the skeleton, ‘teeth and skin to a scientific institution in Dallas, Texas. What did the scientists probably do with the skeleton, teeth and bones? A. Sent them to a place with more experienced scientists. B. Kept them for souvenirs. C. Studied them carefully and compared them to other animals D. Sold them to museums that displayed them D. Directions: Read the following passage and answer the question. Pretend you are at the grocery store..As you are walking down the cookie aisle, you notice a little .girl crying and pulling on her mom's coat .as she pushes the cart towards the front of the store. The closer she gets to the front of the store, the louder the little. girl:screams. Lots of things could be wrong with the little girl, BUT we can probably conclude that- Which of the following statements can be inferred from the paragraph? A. The little girl is very tired. B. The little girl is hungry. C..She wants cookies, and her mom said no. D. She wants her father. E. Directions: Read the following passages and answer the questions that follow. The best advice from professionals is directed toward parents. Therapists encourage parents to avoid situations that usually end in power struggles, to try not to feed into oppositional behavior by reacting emotionally, to praise positive behaviors, and to discourage negative behaviors with timeouts instead of harsh discipline. Which of the following statements can be inferred from-the above paragraph? A. Parents of children with ODD are bad parents. . ODD is not a real psychological disorder. Mysterious and forbidding . Medication can worsen ODD. . Reacting emotionally to defiant behavior might worsen the behavior. moom 87 On a bad day, have you ever been irritable? Have you ever used a harsh tone or even been verbally disrespectful to your parents or teachers? Everyone has a short temper from time to time, but current statistics indicate that between 16% and 20% of a-schoo!'s population suffers from a psychological condition known as oppositional defiant disorder, or ODD. Which of the following can be inferred from the above paragraph? A. Most children who speak harshly to their parents have ODD. B. Most people exhibit symptoms of ODD occasionally. C. Between 16% and 20% of the school population has been abused. D. A short temper is a symptom of obsessive compulsive disorder. EXERCISE 2 Directions: Using your best inference’ strategies, make an inference about the following statements. The first two are done for you. 4. Lwouldn't eat after that two-year-old if | were you. Inference: The two-year-old probably did something gross to the food you were about to eat or has a cold and you could catch it. Something bad will happen to you if you eat the food. 2. For Valentine’s Day, my fantastic neighbor gave his wife a poem that took him about two seconds to write. Sheesh. Inference: My neighbor is not very considerate since he didn't take his time writing the poem. 3. Aman ran after a retreating bus, waving his briefcase frantically. Inference: 4. If she died, | wouldn't go to her funeral. Inference: 5. Jake almost wished that he hadn't listened to the radio. He went to the closet and grabbed his umbrella even though he would feel silly carrying it to the bus stop on such a sunny morning. Inference: Inference A tegical conclusion based lon obaervations. LESSON 4 PRONOUN REFERENTS A pronoun referent is words like he, him, she, her, that, them, one, ones etc. The pronoun that refers to a noun comes after it. Whenever you are asked which noun a pronoun refers to, you should look BEFORE the pronoun to find the noun. Remember the following pronoun referents question: The pronoun “X” in line Y refers to which of the following? Usage - Pronoun Reference . A pronoun is a word used to stand for (or take the place of).a noun. A pronoun should refer clearly to one, clear, unmistakable noun coming before the pronoun. This noun is called the pronoun’s antecedent. Unfortunately, it is very easy to create a sentence that uses a pronoun WITHOUT a clear, unmistakable noun antecedent. Example: After putting the disk in the cabinet, Mabel sold it. The pronoun it does not have a clear noun antecedent. As a result, the reader cannot know for sure whether Mabel sold the disk or the cabinet. The pronoun reference is faulty here because the pronoun it has two antecedents. Such errors, called FAULTY or VAGUE PRONOUN REFERENCE, can confuse readers and obscure the intended meaning. There are three major pronoun reference errors. Error #1; TOO MANY ANTECEDENTS: A pronoun should have only one antecedent. That antecedent should be clear and unmistakable. Look at this sentence: Take the radio out of the car and fix it. Anyone who reads this sentence would not know which item was to be fixed. Does it refer to the radio or the car? The answer is unclear. 89 In the above example, faulty pronoun reference occurs because the pronoun it has two possible noun antecedents. To fix the sentence, substitute a noun for thie pronoun. ‘Take the radio out of the car and fix the radio. OR Take the radio out of the car and fix the car. Here is another example of faulty pronoun reference caused by more than one noun antecedent: The supervisors told the workers that they would receive a bonus. | The pronoun reference is unclear: Who will get the bonus - the supervisors or the workers? They could refer to either group. In this example, the best way to fix the pronoun reference problem is to rephrase the sentence. Revision #1 (gives the bonus to ‘the workers) The supervisors complimented the workers on receiving a bonus. Revision #2 (gives the bonus to the workers) The supervisors told the workers to expect a bonus. Revision #3 (gives the bonus to the supervisors) The supervisors told the workers that they themselves were expecting a bonus. Error #2: HIDDEN ANTECEDENTS Faulty pronoun reference errors also occur when the pronoun's antecedent functions as an adjective rather than a noun. In such cases, the true antecedent is "hidden" or obscured from the reader because it has been subordinated to another noun. The candy dish was empty, but we were tired of eating it anyway. The reader of this sentence might think that the dish was being eaten because dish appears to be the antecedent for the pronoun it. Example: Obviously, people do not eat dishes. What this writer means to say is, "We were tired of eating candy." However, ‘candy cannot be the antecedent for it because candy, situated in front of the noun dish, is acting like an adjective. Only nouns can be antecedents. To fix the sentence, substitute a noun for the pronoun it. The candy dish was empty, but we were tired of eating candy anyway. Here is another example of faulty pronoun reference caused by a hidden antecedent: Mark called Mary's house all day, but she never answered the phone. Obviously, she refers to Mary since a house would NOT be able to answer a phone. However, Mary's modifies house - Mary's is a hidden antecedent and, thus, is not clear. To repair this error, we can change the pronoun = to anoun. Mark called Mary's house all day, but Mary never answered the phonic. Another way to repair this error is to remove the hidden antecedent. Mark called Mary all day, but She never answered the phone. 91 ‘Still another way to repair this error is to rephrase the sentence. Mary never answered the phone, although Mark called her house all day. Error #3: NO ANTECEDENT AT ALL Another kind of faulty / vague pronoun reference problem occurs when writers use a pronoun without giving the pronoun any antecedent at all. Example: The witness called the television station, but they didn't answer. In this example, the pronoun they has NO noun antecedent to which it can refer. To repair this error, we change the pronoun they to a noun. “The witness called the television station, but the reporters didn't answer Another way to repair this error is to create an antecedent - one that is clear and unmistakable The witness called the television reporters, but they didn't answer. Here is another example of a pronoun without any antecedent at all. Although Mrs. Smith was wealthy, she made poor use of it. In this example, the pronoun it has no antecedent to which it can refer. The reader knows that Mrs. Smith is “wealthy,” but it cannot refer to wealthy because wealthy is not a noun. There are at least two ways to repair this error. Replace the pronoun it with anoun. Although Mrs. Smith was wealthy, she made poor use of her wealth. 92 With a noun (wealth).in the place of the pronoun (it), no antecedent is needed. Rephrase the sentence so that the first part contains an antecedent for the pronoun it. Although Mrs. Smith had a lot of money, she made poor use of Now the pronoun it has a clear noun antecedent: money. Here is another example of a pronoun without any antecedent. it says in the paper that the legislation was passed. It, which appears at the very beginning of the sentence, has no noun antecedent at all. In addition, the construction /t says in the paper is unnecessarily wordy. We can repair this error by writing a more DIRECT version of “It says in the paper." Example: The paper says that the legislation was passed. Another way to repair the “It says in the paper” error is to rephrase this part of the sentence. Example: According to the paper, the legislation was passed Both methods of repairing this faulty/vague pronoun error eliminate the pronoun and, thus, eliminate the need for an antecedent. Below, another example shows how this error in pronoun reference occurs when a pronoun is used to stand for (refer to) a whole group of words INSTEAD OF one clear noun antecedent. | did not attend the rally, which was very unpatri of me The word which has no single, clear antecedent. Instead, it refers to the entire clause - "I did not attend the rally.” 93 However, a pronoun must always refer to a single, clear, unmistakable NOUN ANTECEDENT. We can repair this error in at least two ways. 1. Replace the pronoun which with a noun. | did not attend the rally. My actions were very unpatriotic 2. Rephrase the sentence to eliminate the pronoun. By not attending the rally, | was unpatriotic. OR Because | did not attend the rally, | was very unpatriotic. OR | My riot attending the rally was very unpatriotic. OR Not.attending the rally was very unpatriotic of me. Here is another example of faulty pronoun reference where a pronoun is asked to refer to a whole group of words instead of a clear, single noun antecedent. Meg telephoned Howard yesterday to explain why she had not attended the meeting the day before. This made Howard angry. The problem here is This. Its antecedent is the entire preceding sentence. The reader cannot be sure whether Howard is very angry because: 1, Meg telephoned, 2. Meg telephoned yesterday, or 3. Meg had not attended the meeting the day before. There are at least two ways to repair this error and create a clear antecedent for this: 1. Replace the pronoun this with anoun. Meg telephoned Howard . .. before: . Meg's absence made Meg's late cali made ... 2. Rephrase the sentence to eliminate the pronoun: Because Meg telephoned Howard yesterday to explain why she had not attended the meeting the day. before, Howard became very angry. : OR ‘Meg’s telephone call yesterday to'explain why she had not attended thé meeting the day before made Howard very angry: Note: Watch out for "this" and “which" pronouns. Often they are used incorrectly and create faulty or vague pronoun reference problems. Exercise: Directions: Read each of the following sentences for faulty pronoun reference. What is wrong with each of them? 4. When Joshua put the stapler on the chair, it fell. 2. They said on the news that the storm is heading towards the region. 3.' Ihave always wanted to work in a pet store, and last month I did it 4. It says on the bottle to take two tablets with meals. 5. Tom has recently divorced his wife, which makes him depressed. 6. Jane's telephone rang seventeen times, but she didn't answer. 7. The heiress lived a life of luxury. This did not make her happy. 8. Take the cupcakes from the children and eat them. 9. In Hemingway's novels, he uses simple words and sentences. 10. In the book it says that the war was won after a long, bloody battle. 95; Directions: Read the following and answer the questions that follow. EXERCISE 1 The full moon that occurs nearest the equinox of the Sun has become known as the Harvest Moon. It is a bright moon which allows farmers to work late into the night for several nights; they can work when the moon is at its brightest to bring in the fall harvest. The Harvest Moon of course occurs at different times of the year ion the Northem and Southern Hemispheres. In the Northern Hemisphere, the Harvest Moon occurs in September at the time of the autumnal equinox. In the Southern Hemisphere, the Harvest Moon occurs in March at the time of the vernal equinox. 1. The pronoun “It’ in line 2 refers to A. the equinox C. the Sun B. the Harvest'Moon: D. the-night 2. The pronoun “they’ in line 3 refers to A. farmers C. nights B. times of the year D. Northern and Southern Hemispheres EXERCISE 2 Mardi Grass, which means “Fat Tuesday” in French, was introduced to America by French colonists in the early eighteenth century. From that time it has grown in popularity, particularly in New Orleans, and today it is actually a legal holiday in several southern states. The Mardi Grass celebration in New Orleans begins well before the actual Mardi Grass Day. Parades, parties, balls, and numerous festivities take place throughout the week before Mai Grass Day; tourists from various countries throughout the world flock to New Orleans for the celebration, where they take part in a week of nonstop activities before returning home for some much-needed rest. 1. The pronoun “It” in line 2 refers to A. Mardi Grass C. French B. that time D. New Orleans 2. The pronoun “they” in line 7 refers to A. numerous festivities C. tourists B. various countries D. nonstop activities EXERCISE 3: What do the italic words refer to? The operation of a social group depends on its members having certain parts, or roles, to play. Each member has a particular position in the structure of the group and has particular tasks to perform in its activity. So his part carries with it certain obligations. But it also gives him certain rights, since the performance of his tasks entitles him to receive services in return from other members of the group. These sets of rights and obligations can be said to define the individual's social roles. 96 Now answer ihe following. Many universal features of human languages seem both complex and arbitrary. It would not be difficult to imagine simpler ‘systems of vocal'communication, systems that one would expect to Line _ be easier for children to learn. But that misses the point. An adult's (5) language has the features it does precisely because these features people find most natural and easy to learn as young children. The problem, therefore, is not to understand how a child acquires language in spite of its complexities but rather to understand how the complexities of adult language are shaped by (10) innate predispositions of the child. Viewed in these terms, searching for the child’s innate abilitiés to process language is an obvious strategy for research. Children the world over pass through a series of similar stages on their way to becoming fluent speakers of their native tongue. (15) The acquisition of language proceeds in a regular sequence; some Children pass through more rapidly than others, but the sequence is invariant. If all children lived in a ‘world that offered them language in the same order, unfolding progressively the way a good textbook is written, the ‘regularity of development could be explained in terms of regularity of experience. Most students of child language, however, find it more plausible to attribute this regularity not to uniform experience “but ‘to the ‘regularity of biological processes of maturation. 01. The word “its” in line 8 réfers to (A) child (C) language (B) predisposition (D) problem 02. The phrase “the world over” in line 12 is closest in meaning to (A) in large numbers (C) in important places (B) everywhere (D) above ground 03. The word “others” in line 15 means other (A) abilities (C) children (B) fluent speakers (D) stages 04. What is the author's main purpose in the passage? (A) To persuade the reader to investigate a variety of human languages (B) To outline the various complexities found in human languages (C) To advance the view that acquiring a first language is a part of biological maturation (0) To criticize the view that children acquire language in uniform stages 97 READING COMPREHENSION PRACTICE TEST Directions In this, section you will read several passages. Each one is followed by several questions. You are to choose the one best answer, A, B, C or D, to each question. : ‘ Answer all questions about the information in a passage on the basis of what is stated or implied in that passage. Questions 1-3 A new atomic clock being developed for navigation satellites will perform better than previous devices. The clock will use a new microwave cavity design to provide a compact and lightweight package and new electronic techniques to maintain long-term stability. The clock can provide precise navigation information because it is stable to one second in three million years. The differences in the time when signals from four satellites arrive at one location can be used to calculate that position to within a few yards. 1. .A. It can be inferred from the passage that the new clock will be long- lasting B. harmful to humans C. produced in great numbers. D. very attractive looking 2. According to the passage, signals from how many satellites will be used to calculate a position? Ad B.2 C.3 D.4 3. Whatis the primary purpose of the passage? A. To teach a lesson B. To sell a product C. To support a theory D. To provide information Questions 4-5 Joy and sadness are experienced by people in all cultures around the world, but how can we tell when other people are happy or despondent? It tums out that the expression of many emotions may be universal. Smiling is apparently a universal sign of friendliness and approval. Baring the teeth in a hostile way, as noted by Charles Darwin in the nineteenth century, may be a universal sign of anger. As the originator of the theory of evolu- tion, Darwin believed that the universal recognition of facial expressions would have survival value. For example, facial expressions could signal the approach of enemies (or friends) in the absence of language. 98 4. The word despondent in the passage is closest in meaning to ‘A:-Curious B. unhappy : C. thoughtful D. uncertain 5. ‘The ‘author mentions "Baring the teeth in a hostile way" in order to A. differentiate one possible meaning of'a particular facial expression * from other meanings of it. B. support Darwin's theory of evolution. C. provide an example of a facial expression whose meaning is widely understood. D. contrast a facial expression that is easily understood with other facial expressions. Questions 6-8 The Earth's crust is thought:to be divided into huge, movable segments, called plates, which float on a soft plastic layer of rock. Some: mountains were formed as a result of these plates crashing into each other and forcing up the rock at the plate margins. In this process, sedimentary rocks that originally formed on the seabed may be folded upwards to altitudes of more than 26,000 feet. Other mountains ‘may be raised by earthquakes, which fracture the Earth's crust and can displace enough rock to produce block mountains. A third type of mountain’ may be formed as a result of volcanic activity which occurs in regions of active fold mountain belts, such as in the Cascade Range of western North America. The Cascades are made up of lavas and volcanic materials. Many of the peaks are extinct volcanoes. 6. According to paragraph 3, one cause of mountain formation is the A. effect of climatic change on sea level B. slowing down of volcanic activity C. force of Earth's. crustal plates hitting each other D. replacement of sedimentary rock with volcanic rock 7. According to-paragraph 3, one cause of mountain formation is the A. effect of climatic change on sea level B: slowing down of volcanic activity C: force of Earth's crustal plates hitting each other D. replacement of sedimentary rock with volcanic rock 8. According to paragraph 3, one cause of mountain formation is the A. effect of climatic change on sea level B. slowing down of volcanic activity C. force of Earth's crustal plates hitting each other D. replacement of sedimentary rock with volcanic rock Questions 9-11 The railroad was not the first institution to impose regularity on society, or to draw attention to the importance of precise timekeeping. For as lo! iS merchants have set out their wares at daybreak and communal festivities have .been celebrated, people have been in rough agreement with their neighbors as to the time of day. The value of this tradition is today more apparent than ever. Were it not for public acceptance of a single yardstick of time, social life would be unbearably chaotic: the massive daily transfers of goods, services, and information would proceed in fits and starts; the very fabric of modern society would begin to unravel. 9. What is the main idea of the passage? A. In modern society we must make more time for our neighbors. B. The traditions of society are timeless. C. ‘An accepted way of measuring time is essential for the smooth functioning of society. D: Society judges people by the times at which they conduct certain activities. 10. Inline 5, the phrase "this tradition" refers to A. the practice of starting the business day at dawn B. friendly relations between neighbors C. the railroads reliance on time schedules D. people's agreement on the measurement of time 11. In line 8,'the word "very" is closest in meaning to A. much B. extreme C. clear D. actual Questions 12 - 20 The Alaska pipeline starts at the frozen edge of the Arctic Ocean. It stretches southward across the largest and northernmost state in the United States, ending at a remote ice-free seaport village nearly 800 miles from where it begins. It is massive in size and extremely complicated to operate. The steel pipe crosses windswept plains and endless miles of delicate tundra that tops the frozen ground. It weaves through crooked canyons, climbs sheer mountains, plunges over rocky crags, makes its way through thick forests, and passes over or under hundreds of rivers and streams. The pipe is 4 feet in diameter, and up to 2 million barrels (or 84 million gallons) of crude oil can be pumped through it daily. Resting on H-shaped steel racks called "bents," long sections of the pipeline follow a zigzag course high above the frozen earth. Other long sections drop out of sight beneath spongy or rocky ground and return to the surface later on. The pattern of the pipeline's up-and-down route is determined by the often harsh demands of the arctic and subarctic climate, the tortuous lay of the land, and the varied compositions of soil, rock, or permafrost 100 (permanently frozen ground). A little more than half of the pipeline is elevated above the ground. The remainder is buried anywhere from 3 to 12 feet, depending largely upon the type of terrain and the properties of the soil. One of the largest in the world, the pipeline cost approximately $8 billion and is by far the biggest and most expensive construction project ever undertaken by private industry. In fact, no single business could raise that much money, so eight major oil companies formed.a consortium in order to share the costs. Each company controlled oil rights to particular shares of land in the oil fields and paid into the pipeline-construction fund according to the size of its holdings. Today, despite enormous problems of climate, supply shortages, equipment breakdowns, labor disagreements, treacherous terrain, a certain amount of mismanagement, and even theft, the Alaska pipeline has been completed and is operating. 12. The passage primarily discusses the pipeline's A. operating costs B. employees C. consumers. D. construction 13 The word "it" in line 3 refers to A. pipeline B. ocean C. state > D. village 14. According to the passage, 84.million gallons of oil can travel through the pipeline each A. day B.week C. month D. year 15. The phrase “Resting on" in line 11 is closest in meaning to A. consisting of B. supported by C. passing under D. protected with 16. The author mentions all of the following. as important in determining the pipeline's route EXCEPT the A. climate B. lay of the land itself C. local vegetation D. kind of soil and rock 17. The word “undertaken” in line 20 is closest in meaning to A. removed C. transported B. selected D. attempted 101 18. How many companies shared the costs of constructing the pipeline? A. three B. four C. eight D. twelve 19. The word “particular” in line 33 is closest in meaning to A. peculiar B. specific C. exceptional D. equal 20. Which of the following determined what percentage of the construction costs each member of the consortium would pay? A. How much oil field land'each company owned B. How long each company had owned land in the oil fields C. How many people worked for each company D. How many oil wells were located on the company's land Questions 21 - 22 Archeology delivers traces of dance from prehistoric times such as the 9,000 year old Bhimbetka rock shelters paintings in India and Egyptian tomb paintings depicting dancing figures from 3300 BC. One of the earliest structured uses of dances may have been in the performance and in the telling of myths. Before the production of written languages, dance was one of the methods of passing these stories down from generation to generation. Another early use of dance may have been as a precursor to ecstatic trance states in healing rituals. Dance is still used for this purpose by many cultures from the Brazilian rain forest to the Kalahari Desert. Some European tribes also used loud group singing and dancing in order to prepare themselves for the dangerous combat with other tribes. 21. According to paragraph one, why was dance used to pass stories down between generations? A. Because it was more expressive than writing B. Because writing had not been developed C. Itwas a good way of depicting stories from myth D. Dances could be shared between tribes 22. Which.of the following is not listed as a historic use of dance in paragraph 17 A. The telling of myths B. Healing rituals C. As a prayer for good weather D. To prepare for combat 102 Questions 23 - 24 By the 18th century ballet had migrated from the royal court to the Paris Opera. During this century the ballet spread through Europe and developed from a courtly arrangement of moving images-used: as part of a larger spectacle, to a performance art in its own right, the ballet d'action. This new form swept away much of. the artificiality of the court dance and strove towards the concept that art should aspire to imitate nature. This ultimately resulted in costumes that allowed the dancer. much more freedom of movement than before and were conducive to a fuller use of the expressive capacity of the body. It also opened the door to pointework, for this acceptance of more naturalistic costuming allowed the development. of the heel-less shoe, which led to the dancer being able to make more use of the tise onto demi-pointe. - 23. What does the author imply about ballet costumes? A. Costumes used in the 17th century often restricted the movements of ballet dancers: B. Early ballet costumes were modeled after the appearances of animals in nature C. Dancers performed barefoot on stage D. Early ballet costumes differed between European countries 24. In line 3, the word “courtly” is closest in meaning to A. process B. creative C. very polite D. partly Questions 25 — 26. A brief exception tothe. new ideas and designs used in the-twentieth century occurred: in. ‘the :1960's,- which saw the growth of postmodemism. Postmodernism veered towards simplicity, the beauty of small things; the beauty of untrained body, and-unsophisticated movement. The famous 'No’ manifesto rejecting all costumes, stories and outer trappings in favor of raw and unpolished movement was perhaps the extreme of this wave of thinking. Unfortunately lack of costumes, stories and outer trappings do not make a good'dance show, and by the end:of the 1960s, sets, décor and shock value re-entered the vocabulary of modern choreographers.” 25. Why does the author mention the "No” manifesto? A.To explain the cause of minimalistic dance perfomances in 1960s dance B. To show the effect that new ideas had on dance performances in the 1960s C. To illustrate the extreme ideas that some dancers and choreographers had in the 1960s D. To explain why dance performances after the 1960s once again used sets, decor and shock value 103 26. Why does the author mention that performances without costumes and stories don't result in good shows? A. To explain why dance performances after the 1960s once again used sets, decor and shock value B. To criticize dance performances of the 1960s C.To compare dance performances of the 1960s to those after the 1960s. D. To account for trends in 1970s ballet Questions 27 The 20th century was indeed a period of breaking away from everything that ballet stood for: It was’a time of unprecedented creative growth, for dancers and choreographers. It was also a time of shock, surprise and broadening of minds for the public, in terms of their definitions of what dance was. It was a revolution in the truest sense. 27. The word "unprecedented" is most similar in meaning to: A. unique B. popular C. demanding D. difficult Questions 27 - 35 Most people would say that the world’s tallest mountain is Mount Everest. This mountain in the Himalayas is just over 29,000 feet high. However, if mountains are measured a little bit differently, then the tallest mountain on Earth is Mauna Kea, in the Hawaiian Islands. Mauna Kea is only about 14,000 feet above sea level, so in comparison to Mount Everest it just does not look anywhere near as high as Mount Everest to a person standing at sea level. Mauna Kea, however, does not begin at sea level. It rises from an ocean floor that is more than 16,000 feet below the surface of the water. This mountain therefore measures more than 30,000 feet from it base to its top, making it a higher mountain than Mount Everest. 28. The main idea of the passage is that ‘A. Mount Everest is the world’s tallest mountain B. Mount Everest and Mauna. Kea are located in different parts of the world C. Mauna Kea’s base is below sea level D. Mauna Kea could be considered the tallest mountain in the world 29. Which of the following is NOT stated about Mount Everest? A. Many people believe it is the world’s tallest mountain. B. It is part of the Himalayas. C. It is over-29,000 feet high. D. It rises from the ocean floor. 30. The word “just” could best be replaced by A. noticeably B. soon C. barely _D. recently 104 31. The expression “a little bit” is closest in meaning to A.asmall size B. quite C. somewhat D. extremely 32. Aecorng to the passage, Mauna Kea is how far above the level of the water? A. 14,000 feet B. 16,000 feet C. 29,000 feet D. 30,000 feet 33. The expression ‘in comparison to” could best be replaced by A. close to B. in relation to C. as a result of D. because of 34. Itis implied in the passage that Mauna Kea does not seem as tall as Mount Everest because A. people do not want to look at it B. part of Mauna Kea is under water C. Mount Everest has more D. snow Mauna Kea is in a different part of the world than Mount Everest 35, The word “floor” is closest in meaning to A. carpet B. bottom C. roof D. water Questions 36 - 43 Coce-Cola was invented in 1886 by Atlanta pharmacist John S. Pemberton, The name for the product was actually proposed by Pemberton's assistant, Frank Robinson. The name was taken from the two most unusual ingredients in the drink, the South American coca leaf and the African cola nut. The recipe for today’s Coca-Cola is very well guarded. Many of the ingredients are known; in addition to coca leaves and cola nut, they include lemon, orange, lime, cinnamon, nuimeg, vanilla, caramel, and sugar. The proportions of the ingredients and the identity of Coke secret ingredients are known by only a few of the Coca-Cola Company's senior corporate officers. 36. The passage mainly discusses A. the success of the Coca-Cola Company B, the unusual ingredients in Coca-Cola C. John S. Pemberton D. Coca-cola’s recipe and who developed it 37. According to the passage, who created Coca-Cola? A. The Coca-Cola Company C. John S. Pemberton B. Pemberton’s assistant D. Frank Robinson 38. The word “unusual” is closest in meaning to A. uncommon B. important C. unused D. tasty 105 39. Which of the following is NOT true about the name Coca-Cola? A. The name “coca” comes from the coca leaf. B. The name “cola” comes from the cola nut. C. Frank Robinson suggested the name. D. The inventor came up with the name. 40. A ‘recipe’ is A. information about drugs fora. pharmacy B. a description of how to prepare something C. an accounting statement D. a corporate organizational plan 41. The passage states that the recipe for Coca-Cola is A. well known, C. unknown B. known by onlya limited number of people —_D. published information 42. Which of the following is NOT mentioned as an ingredient of Coca-Cola? A. Orange leaves B.Nutmeg — C. Citrus fruits D. Sugar 43. The word “secret” could best be replaced by A. Unrevealed B. delicious C. business D. speechless Questions 44-50 When Columbus arrived in the Americas in 1492, there were already an estimated thirty to forty milion people living in North and South America. It has therefore been quite easy for some to refute the idi that Columbus “discovered” ‘America. How and when these inhabitants came to ‘America has been the source of much scientfic research and discussion. Most archeologists agree that the first ‘Americans the tue “discoverers” of America,came from northeastem Asia. There is also a considerable amount of proof that inhabitants have been in the Americas for at least 15,000 years. To get to the ‘Americas, these people had to cross over the 55-mile-wide Bering Strait that separates ‘Asia and North America. According to one theory, these people crossed over during periods when a land bridge existed between the two continents. During Ice Ages, so much of the Earth's water was frozen that the sea levels dropped, and it was possible to walk from Asia to North America, 44, Whatis the author's main purpose? A. To explain how Columbus discovered America B. To show how people came to America before Columbus C. To demonstrate the importance to archeologists of northeastem Asia D. To explain how to cross the Bering Strait 45. In 1492, how many people were probably in the Americas? A. Fewer than thity milion C. Forty milion or fewer B. Exactly thirty milion D. Atleast forty milion 34. 106 46. 47. 48. 49. 49. The word “refute” is closest in meaning to A. theorize B. support C. contradict D. defend It is implied in the passage that A. Columbus was really the first person in America B. scientists are sure about America’s first inhabitants C. Columbus arrived at almost the same time as America’s first inhabitants D. all is not known about America's first inhabitants 36. There is general agreement that the first people who came to North America came from A. B. South America C. northeastem Asia D. Africa 37. The word “considerable” could best be replaced by which of the following? A. Large B. Weak C. Well-known D. Considerate ‘The word “inhabitants” is closest | meaning to . inhibits . fish . residents . blocks pOm> 107

You might also like